Torts Multiple Choice

Ace your homework & exams now with Quizwiz!

A woman suffered extensive chemical burning to her scalp when her hairdresser used a defective batch of hair dye. The woman spoke to her lawyer, who explained to her that there were several courses of action available. Upon advice of her lawyer, the woman decided to bring a products liability suit against the hairdresser, but not the manufacturer of the hair dye. Under what theory or theories of products liability may the woman sue the hairdresser? A. A negligence theory of recovery in a products liability action only. B. A strict liability theory of recovery in a products liability action only. C. A negligence theory and strict liability theory of recovery in a products liability action. D. A pure negligence action only, as products liability is not applicable in this case.

A. A negligence theory of recovery in a products liability action only.

A 16-year-old student took his date for the high school prom out to dinner before the dance. It was the first time the student had ever been to a restaurant without his parents, and the first time he had ever been to such an expensive restaurant. The student had badly underestimated the amount that dinner would cost, especially because his date ordered one of the most expensive entrees. He only had the change in his pocket left to tip the waiter after paying for their meal. Their waiter was a particularly belligerent person who, when he saw that the student had left a few coins for his tip, grabbed the coins from the table and threw them in the student's face, shouting, "You miserable, pimple-faced bumpkin! How dare you insult me by leaving this pittance as my gratuity?" When the student attempted to whisper an explanation, the waiter continued loudly, as everyone in the crowded restaurant looked on, "I don't want to hear your protestations of poverty. I could tell from your grotesque manners and from the way you're dressed that you are an uncouth, unlettered ingrate." When the student attempted to leave, the waiter pushed him roughly to the floor, exclaiming, "Pick up your ridiculous money!" The student grabbed at the coins on the floor and fled, humiliated. Unknown to the student, his mother had booked a table in a secluded portion of the same restaurant so that she could videotape his first date. She observed the entire episode with the waiter, suffering severe emotional distress and as a consequence, subsequently brought suit against the waiter for intentional infliction of emotional distress. What result is likely? A. Judgment for the waiter, because he was unaware of the mother's presence in the restaurant. B. Judgment for the waiter, because the mother was not close enough to have been threatened with any physical harm from the waiter's conduct. C. Judgment for the mother, because she suffered severe emotional distress as result of watching the waiter humiliate her son. D. Judgment for the mother, if the waiter's conduct was extreme and outrageous.

A. Judgment for the waiter, because he was unaware of the mother's presence in the restaurant.

A famous Las Vegas hotel and casino owner carefully selected an independent contractor to renovate and upgrade his nightly water fountain show. The hotel was open for business during the renovations. One day, while walking past the fountain area, a hotel guest fell on a large piece of pipe that the contractor had negligently left exposed. The guest's leg was injured as a result of the fall. The guest filed suit against the hotel owner to recover for his injuries. Who should prevail? A. The guest, because the owner owed a non-delegable duty to the guest to keep the premises safe. B. The guest, because he was injured on hotel premises. C. The owner, because the independent contractor was the legal cause of the guest's injuries. D. The owner, because he exercised reasonable care in selecting the independent contractor.

A. The guest, because the owner owed a non-delegable duty to the guest to keep the premises safe.

One night, a man and woman were waiting at a crowded train station to board the train to a baseball game. The crowd was rowdy and excited because the winner of the game would be crowned the division champion. When the train arrived, an excited fan, who was carrying a large foam finger and a heavy backpack, pushed in front of the couple. As the man struggled to board the train, the fan dropped his backpack on the man's foot. The man heard an explosion, felt pain in his foot and fell to the floor of the train. Because the man suffered from osteoporosis, the weight of the backpack broke ten bones in the man's foot. When the police arrived, they discovered that the fan had been carrying a bag of explosive fireworks in the backpack that he planned to detonate in a big celebration after the game. The jurisdiction had enacted an ordinance prohibiting the carrying and detonation of all explosive fireworks. The man filed a lawsuit against the fan to recover his medical expenses. In his complaint, the man alleged negligence per se. Which of the following statements is most accurate? A. The man will not recover, because the harm was not of the type that the statute was designed to prevent. B. The man will not recover, because his injuries were unforeseeably extensive. C. The man will recover, because the backpack was heavy due to the fireworks, and the man was within the class of people the statute was designed to protect. D. The man will recover, because the man can prove causation and damages.

A. The man will not recover, because the harm was not of the type that the statute was designed to prevent.

A landlord in a large city owned property that bordered a local dead end street. There were no houses on the dead end street so traffic was very light. In fact, the street was often used as an area for block parties and for neighbors to wash their cars. A large 100-year-old elm tree on the landlord's property hung over the sidewalk beside the street. During a windstorm a large branch broke off and injured a boy who was riding his bicycle on the sidewalk without his mother's permission. The landlord admitted that he never had the tree inspected or serviced by an arborist. The boy's parents subsequently brought an action on behalf of their son against the landlord. Should the boy's parents prevail? A. Yes, because the landlord owed a reasonable duty of care to people on the sidewalk. B. Yes, because the homeowner owed a nondelegable duty to people outside the premises. C. No, because the boy assumed the risk by riding his bicycle under the tree limb. D. No, because the boy was an unknown trespasser.

A. Yes, because the landlord owed a reasonable duty of care to people on the sidewalk.

A woman enters a church in order to pray for spiritual guidance. On her way out, she trips on a crack in the floor and falls. She now seeks to sue the church for damages under a claim of negligence. In a majority of jurisdictions, how will the woman be classified? A. Business invitee. B. Public invitee. C. Licensee. D. Known trespasser.

B. Public invitee.

A businessman had just sold a grocery store to a competitor and was feeling a bit bored. He liked the food business and so he decided to open a restaurant in his home town. He purchased all the necessary furniture and hired the needed staff. Two weeks after the opening, the restaurant received a new shipment of beef to serve to the customers. The chef then cooked the meat. A waiter then served some of the beef that had just been cooked to an elderly woman who had a weakened immune system. The elderly woman ate some of the beef, but then became very sick and had to go to the hospital. The doctor in the hospital concluded that the woman became sick because the beef had some bacteria that are known to cause illness in humans. The woman filed a strict product liability suit against the restaurant. Which of the following is correct? A. The woman will prevail, because she had a weakened immune system. B. The woman will prevail, because the restaurant is a seller of goods. C. The restaurant will prevail, because it is only a service provider. D. The restaurant will prevail, because the chef cooked the meat that was served to the woman.

B. The woman will prevail, because the restaurant is a seller of goods.

In a nuisance case, it is a defense that the plaintiff "came to the nuisance" (that is, the purported nuisance was present before the plaintiff (or the plaintiff's predecessor in interest) began using the land). A.True B.False

B.False

Darrin is angry at Pedro. He decides to kill him. Pedro is standing on the street corner. Darrin is driving, about a block away. Spotting Pedro, Darrin accelerates his car to 80 mph aiming directly for Pedro. Fortunately, he narrowly misses hitting Pedro, zooming past within inches. Less fortunately, Pedro, startled by the near miss, has a heart attack and dies. Darrin's MOTIVE is: A. To scare Pedro. B. To kill Pedro. C. Anger at Pedro. D. A jury question that cannot be determined from the facts given.

C. Anger at Pedro.

A bank customer had just finished withdrawing funds from an ATM when a robber approached, pulled out a knife, and ordered the bank customer to hand over all of his money. Instead, the bank customer turned, and the robber ran after him in hot pursuit, waving the knife. The bank customer ran into a yard and picked up a hoe left in the yard by a gardener who was doing his Saturday morning maintenance. The robber was momentarily deterred. During that moment, the gardener returned, and seeing the robber apparently perilously close to decapitation by the hoe, seized the bank customer's arms from behind and wrenched loose the hoe. The gardener's action dislocated the bank customer's shoulder. If the bank customer sues the gardener, will he prevail? A. Yes, because the gardener intended to cause a harmful touching. B. Yes, because the gardener assumed the risk that his actions were unreasonable under the circumstances. C. No, because the gardener was acting in defense of another. D. No, because there was a public necessity for the gardener's actions.

C. No, because the gardener was acting in defense of another.

A home builder purchased a large plot of land that had recently been zoned for residential use. He subdivided the land into 10 lots and built one house on each lot. The home builder easily sold the houses on Lots 1 though 6. However, a terrorist attack caused a major downturn in the economy, and the home builder could not sell the remaining lots. The home builder sold Lots 7 through 10 to a real estate agency. The agency then sold Lot 7 to a woman that ran a daycare in her home. The woman owned the house for about three months before she noticed a number of defects in the construction. Several of these defects caused the woman and the children she cared for to become injured. The woman seeks to bring a strict products liability action with respect to these defects. Will the woman be able to bring a strict products liability suit against the real estate agency? A. Yes, because the real estate agency was in the distribution chain of the house. B. Yes, because the real estate agency sold the house to the buyer as part of mass-marketed new residences. C. No, because the real estate agency did not construct the house. D. No, because strict products liability does not apply to the sale of real property.

C. No, because the real estate agency did not construct the house.

The local authorities concluded that Six Sixty-Sixth Street is the headquarters of a dangerous organized crime gang. In the middle of the night, a fully equipped SWAT team pulled up in front of the house with a military-surplus tank that the Pentagon donated to the police force. The police proceeded to blast the door open and invade the house in a manner consciously reminiscent of the famous videos of the capture of Bin Laden. However, no one was home, and no drugs or evidence of crime were found. The house was totally destroyed. It later turned out that the raid had been based on a tip from a paid informant who had recently binge-watched a series of shows on Satanism and zombie body-snatchers while seriously stoned. Pedro, the owner of the building, sues the City for damages. The City has waived sovereign immunity and does not challenge that is responsible for the actions of its employees and agents, including the relevant detectives, the SWAT team and those who planned the raid. A. The City is liable because it has caused damage by intentionally entering upon real property owned by another. B. The City is liable providing that Pedro can establish that relying on the tip, or the manner in which the raid was performed, was not reasonable under the circumstances. C. The City is liable for the damage to Pedro's property. Regardless of whether it was privileged to conduct the raid, it is not entitled to do so without compensating him for the damage it did. D. The City is not liable if the raid was ordered in bona fide pursuit of the public interest, even if the decisionmakers erred. E. The City is not liable because Pedro's house, even if it is his home, is not part of his person, and touching it -- even destroying it -- is not an injury to Pedro's person.

C. The City is liable for the damage to Pedro's property. Regardless of whether it was privileged to conduct the raid, it is not entitled to do so without compensating him for the damage it did.

A mechanic stopped making loan payments on his six-month-old car. The dealership, who also made the loan, intended to repossess the mechanic's car. The tow truck driver, however, towed away a dentist's car. The mistake was understandable because the car was the exact same year, make, model and color as the mechanic's car. After four hours, the tow truck driver discovered his mistake and returned the dentist's car. The dentist filed suit against the tow truck driver for damages. Who should prevail? A. The tow truck driver, because the damages were nominal. B. The tow truck driver, because he made an honest mistake. C. The dentist, under the theory of trespass to chattels. D. The dentist, under the theory of conversion.

C. The dentist, under the theory of trespass to chattels.

A golfer was playing his friend for $10 a hole. On the second hole, the golfer hit an errant shot that bounced off a sprinkler head on the golf course. The golf ball landed on a rancher's land that bordered the golf course. The golfer then walked onto the rancher's land to retrieve his $3 golf ball. Which of the following statements is most accurate? A. The golfer is liable for trespass to land for hitting the ball on to the rancher's land, but not for retrieving his ball. B. The golfer is liable for trespass to land for hitting the ball on to the rancher's land and also for retrieving his ball. C. The golfer is not liable for trespass to land for hitting the ball on to the rancher's land, but is liable for retrieving his ball. D. The golfer is not liable for any of his actions.

C. The golfer is not liable for trespass to land for hitting the ball on to the rancher's land, but is liable for retrieving his ball.

A woman decided to have a birthday party in her apartment complex for her three-year-old. She invited the neighbors and a few close friends. While playing in the bedroom, a young boy was bitten on his face by the woman's dog. The injured child's mother was socializing with her neighbors and did not see the incident. Before moving to the apartment, the dog had only bitten one other child, but no medical treatment was required. The young boy required several stitches near his eye but the bite would not likely leave a permanent scar. The young boy's mother filed suit against to party host. Who should prevail? A. The host, because she did not have notice of the dog's vicious propensities. B. The host, because the boy's mother failed to supervise her child. C. The guest, because the host was strictly liable for the injuries caused by her dog. D. The guest, because the dog was a non-obvious danger.

C. The guest, because the host was strictly liable for the injuries caused by her dog.

Tam wants to build a beautiful new home on a coastal beach barrier island connected to the mainland by a low bridge. Her mortgage lender insists that she have flood insurance because it is concerned that, in the event of flood damage, she will not have enough money to both repair the home (which is the bank's security for the loan) and pay back the loan. When she goes to buy flood insurance, her homeowner's insurance carrier declines to sell it, citing the difficult of predicting floods in a period in which the national flood maps are known to be out of date and the relevant government agency is headed by a climate change denier. She is referred to a high-risk carrier which wants to charge her 10% of the value of her home each year. While the national flood maps predict that her new home might be flooded every 20 years or so, some forecasters believe that the increase in sea levels and more virulent storms due to warmer seas mean that floods will be far worse than the old predictions and that there is even a 20% chance that the entire barrier island will be uninhabitable within the term of Tam's mortgage. Which of the following is most accurate? A. The high-risk insurance company is taking advantage of Tam's lack of market power to charge her a price that should not exist in a competitive market. It would be appropriate for regulators force it to charge less. B. The high cost of insurance is a market failure that can only be solved by recreating the national flood insurance plan, which would prevent discrimination against homeowners in flood zones. C. The high cost of insurance for Tam reflects the high cost of living in flood zones, where rescue, repair and rebuilding are highly predictable charges that insurers must include when setting their rates. Any insurer that charges less than the claims than it will predictably have to pay out will be unable to remain solvent, absent tax-payer financed subsidies (such as the repeated bailouts of the Federal flood insurance system). If society wishes to subsidize Tam's desire to live on the beach, it should probably do so in a transparent way that allows voters, politicians and Tam to see the subsidy and consider whether it is the best use of our collective resources. D. Tam's decision to build on her own property is a personal freedom that should not be subject to regulation by big government.

C. The high cost of insurance for Tam reflects the high cost of living in flood zones, where rescue, repair and rebuilding are highly predictable charges that insurers must include when setting their rates. Any insurer that charges less than the claims than it will predictably have to pay out will be unable to remain solvent, absent tax-payer financed subsidies (such as the repeated bailouts of the Federal flood insurance system). If society wishes to subsidize Tam's desire to live on the beach, it should probably do so in a transparent way that allows voters, politicians and Tam to see the subsidy and consider whether it is the best use of our collective resources.

George purchased a new pickup truck from dealer for his pizza delivery business. As a result of a defective crankshaft in the motor, George cannot drive the truck. George filed a lawsuit against the dealer and the manufacturer. At trial, George proved that he suffered $10,000 in lost profits. The defendants filed a motion to dismiss the case. How should the judge rule? A. Dismiss the motion, because George can recover for personal injuries and economic losses. B. Dismiss the motion, because the burden of proving the product was not defective rests with the defendants. C. Grant the motion, because George can only recover for personal injuries. D. Grant the motion, because George cannot recover for purely economic loss.

D. Grant the motion, because George cannot recover for purely economic loss.

A furniture store received a new set of dining room tables and chairs. When the owner of the store looked at the tables, he concluded that the legs of the tables would have to be replaced with more expensive looking legs. He replaced the legs and placed the dining room sets on sale. One week later, a woman was shopping for a new table and chairs for her dining room and saw the furniture store. She purchased one of the new sets and placed it in her dining room. A month later when the woman's family was having a large dinner, the woman placed a very large vase on her table and the table collapsed falling on the legs of the woman's aunt. The aunt filed a strict product liability action against the manufacturer of the table. Which of the following is correct? A. The aunt will prevail, because the table had a manufacturing flaw. B. The aunt will prevail, because the manufacturer is in the chain of distribution. C. The manufacturer will prevail, because the product was sold to the woman, not her aunt. D. The manufacturer will prevail, because the furniture store owner altered the product.

D. The manufacturer will prevail, because the furniture store owner altered the product.

A businessman operates a small child-care facility on his land. He often lets the children run around in the front yard. His neighbor is an owner of a dog, and the neighbor often lets his dog play around in his front yard. Some of the children in the child-care facility are afraid of the dog and they ask their parents to move them to another facility. The businessman has also noticed that many neighbors veer to the other side of the street rather than walk past the dog in the yard. After losing several customers, the businessman decides to sue the neighbor for nuisance. Which of the following is correct? A. The neighbor is liable for damages for public nuisance, because the neighbor's activity substantially and unreasonably interferes with the businessman's use of his land. B. The neighbor is liable for damages for private nuisance, because the neighbor's activity substantially and unreasonably interferes with the businessman's use of his land. C. The neighbor is not liable for damages for public nuisance, because the businessman does not have authority as a public official. D. The neighbor is not liable for damages for private nuisance, because the neighbor's activity is not offensive or inconvenient to the average member of the community.

D. The neighbor is not liable for damages for private nuisance, because the neighbor's activity is not offensive or inconvenient to the average member of the community.

A nurse's aide was employed at an independent living complex for senior citizens. Her employer, the complex owner, was aware that the aide was convicted for stealing jewelry from several residents of a nursing home where she used to work. The owner did not fire the aide because she was an otherwise excellent employee and she claimed that she had been falsely accused of the crime. One evening, the aide was caught in the act of taking jewelry by one of the complex residents, an elderly woman. In order to escape with the items she had stolen from the woman's room, the aide pushed the woman out of her way, causing the woman to fall and break her arm. The woman sued the complex owner for damages. Who should prevail? A. The owner, because the aide was not acting within the scope of her employment. B. The owner, because an employer is not liable for intentional torts committed by an employee. C. The woman, because under the doctrine of respondeat superior the owner is responsible for the actions of her employees. D. The woman, because the owner was aware that the aide had stolen jewelry on past occasions.

D. The woman, because the owner was aware that the aide had stolen jewelry on past occasions.

A homeowner hired the defendant to exterminate rodents living in his house. The defendant took every available precaution to conduct the extermination in a safe manner, but some of the chemicals wafted into the air and sickened the plaintiff, who lived next door to the house that was being exterminated. Will the plaintiff likely succeed in a suit against the defendant for the injuries he suffered when he inhaled the chemicals from the extermination? A. Yes, because the defendant was engaged in an abnormally dangerous activity. B. Yes, under the doctrine of res ipsa loquitor. C. No, because the defendant and the plaintiff were not in privity of contract. D. No, because the defendant was not negligent in conducting the extermination.

A. Yes, because the defendant was engaged in an abnormally dangerous activity.

Insurance -- both private (such as homeowner's liability/fire insurance, no-fault auto accident insurance, or medical insurance) and public (such as worker's compensation, Social Security Administration , Social Security Disability Insurance, flood insurance) -- is sometimes used as a substitute for tort, or to remedy some of the deficiencies of tort (e.g., Liability insurance, especially when it is legally mandated, can protects victims against insolvent tortfeasors). Similarly, publicly funded services can fully or partially substitute for tort: for example, Veterans Administration medical care, Superfund pollution cleanups or Army Corp flood prevention. Which of the following is accurate? A. One goal of both tort and insurance is to spread the cost of injuries, since smaller, predictable, charges are usually less painful than large, unpredictable ones. Tort spreads costs by allowing victims to charge tortfeasors for the wrongful damage they do, assuming they can identify a solvent tortfeasor at fault. If the tortfeasor is a company, it may spread costs by raising its prices (or lowering pay or investor returns), thus making its participants pay for the damages. Or it may insure, in which case the costs ultimately will be borne by all insureds. Unfortunately, insolvent tortfeasors escape liability, and companies may be tempted to create insolvency in order to lower (private) costs. Insurance spreads costs among insurance holders directly. Market pressures, unless the law intervenes, normally mean that private insurance will charge insureds according to the risk they pose to the insurance company, and will redistribute among a risk group from lucky policy holders (who do not suffer injuries) to unlucky ones. Public insurance may be priced differently, but also redistributes from taxpayers or insureds who do not file claims to those who do. Unfortunately, if insurance is not required, insolvent persons may be tempted to go without. B. One goal of both tort and insurance is to create incentives for appropriate levels of safety and care. Tort, by charging people or companies who injure others, gives potential tortfeasors an incentive to invest in safety in order to avoid liability. Unfortunately, this incentive only works if the potential tortfeasor identifies its risk in advance. Additionally, it provides an incentive to hide bad behavior, take advantage of weaker victims who may not be able to assert their rights, invest in aggressive litigation to avoid responsibility, or transfer dangerous activities to insolvent actors. First party insurance -- such as medical, fire and no-fault car insurance -- generally charges according to the likelihood of the insured becoming a victim. Since those at high risk pay more, it creates an incentive for potential victims to visibly reduce the risks they face. Unfortunately, many risks are not reasonably within the victim's control, and first party insurance does not provide incentives for tortfeasor care or safety. Third party insurance, such as liability insurance or worker's comp, charges insureds according to their risk of causing compensable injury, as determined by the insurer. If the insurer is not able to distinguish risk properly, this may result in good actors subsidizing bad actors. C. Tort and insurance can only work if the fact finders are able to make accurate determinations. Tort requires the fact finder to determine difficult issues of intent and fault, and requires the fact finder to make an individualized finding of damages. Thus, it often generates expensive litigation and high legal costs. Insurance determinations are generally made by relatively low-paid bureaucrats on the basis of relatively simple findings of damage and causation, and standard valuations set bureaucratically. If the insurer is run efficiently, this should be a far cheaper, if less individualized, adjudication method. More of the costs of the system, then, go to remedy injuries instead of to intermediaries such as lawyers. D. A and C but not B are correct E. All of A, B, and C are correct.

E. All of A, B, and C are correct.

Under our general law of tort, which of the following is true? A. An actor may hurt another or damage another person's property, without permission or compensation, unless the actor has acted in an unreasonable manner. B. Hurting someone without permission is unreasonable. C. Subsidies distort markets. Accordingly, we do not allow parties to use resources -- including other people's property and services -- with prior agreement and payment. D. Reasonable people pay for the damage they cause to others; tort requires the others to do so as well.

A. An actor may hurt another or damage another person's property, without permission or compensation, unless the actor has acted in an unreasonable manner.

Donald is deluded and believes he has the powers of Midas -- to turn anything he touches into gold. With malice in his heart, he heads to the home of his worst enemy, Peter, intending to turn Peter into gold. He rings Peter's doorbell and says, "Prepare to meet the Goldsmith. Bwa ha ha ha ha!" Peter is terrified -- Donald seems quite deranged and capable of anything. Donald then touches Peter and says "So long sucker." He turns and leaves, imagining that he has turned Peter into a statue. Peter is mystified, but not for long -- he calls his lawyer and tells him to sue. A. Peter probably has a claim for assault but his damages are likely to be minimal. B. Peter probably has a claim for assault, but only if his terror was based on a reasonable belief that Donald was about to do him bodily harm C. Peter does not have a claim in assault because Donald's intentions were irrational. D. Peter does not have a claim in assault because Donald intended to touch him -- and did touch him.

B. Peter probably has a claim for assault, but only if his terror was based on a reasonable belief that Donald was about to do him bodily harm

A hiker was walking on a trail in the woods within the boundaries of a state park. The surface of the road changed as the hiker crossed from the park onto private property but there were no warning signs at the boundary. The hiker came across some rugged terrain and grabbed onto a rock to help pull herself up a hill. The rock dislodged and landed on the hiker's ankle, pinning her down and causing injury to her leg. The hiker did not know that the rock had been dumped by the property owner on unstable soil. The hiker filed an action against the property owner to recover for her injuries. Should the hiker prevail? A. No, because the property owner could not have foreseen that the hiker would be injured by sliding rocks. B. No, because the property owner owed no duty to the hiker. C. Yes, because the property owner had a duty to prevent the rocks on his property from becoming dangerous instrumentalities. D. Yes, because the property owner was liable for hidden dangers on his property.

B. No, because the property owner owed no duty to the hiker.

Without doing a background check, an employer hires a delivery man to make deliveries in local residential neighborhoods. A background check of the delivery man would have revealed that he had a history of causing motor vehicle accidents by driving while under the influence of drugs and alcohol. If the delivery man causes an accident while making a delivery because he is under the influence of drugs or alcohol, can the injured party bring a cause of action based on negligent hiring against the employer? A. Yes, because the cause of action will be based on the doctrine of respondeat superior. B. Yes, because the employer was negligent in failing to conduct a background check on the delivery man before hiring him. C. No, because the employer did not have a duty to investigate the delivery driver's background before hiring him. D. No, because the doctrine of respondeat superior does not apply to intentional torts committed by the employee.

B. Yes, because the employer was negligent in failing to conduct a background check on the delivery man before hiring him.

Frustrated at being denied a big promotion, a man lost his temper and threw a stapler at his boss. However, his boss ducked and the stapler hit one of the man's coworkers in the head. Will the man be liable to his coworker in an action for battery? A. Yes, because the man intended to hit the coworker when he threw the stapler. B. Yes, because the man's intent to hit his boss with the stapler will be transferred to his coworker. C. No, because the man did not intend to hit the coworker with the stapler. D. No, because the doctrine of transferred intent does not apply to intentional torts.

B. Yes, because the man's intent to hit his boss with the stapler will be transferred to his coworker.

A 10-year-old girl was spending the summer with her family at a mountain cabin located on a lake. One afternoon, the girl's parents went to the nearby town to buy supplies, leaving the girl alone at the cabin. As soon as her parents left, the girl ran to her parents' room and took her father's toy motorboat. The girl had frequently watched her father place the toy boat in the lake and operate it with the handheld control panel and wanted to attempt to operate the boat herself, but her father had told her it was too hard for a child to properly operate the controls. The girl decided this would be the day she would operate the toy boat. She ran down to the dock and placed the toy boat in the water. Using the handheld control panel, she started the toy boat's motor and engaged the throttle. The toy boat raced around the lake as the girl steered it, and she kept increasing the speed of the boat until it was going so fast that she was having difficulty controlling it. Suddenly, she noticed that the boat was heading directly for two swimmers, a young couple. The girl tried to steer the boat away from them, but because of the speed of the boat and her inexperience in operating it, she was unable to do so. The toy boat hit one of the swimmers, causing a deep cut in his left arm. He reacted by swinging around to see what had hit him. In doing so, his right elbow hit his girlfriend in the head, causing a deep bruise. If the girlfriend brings suit against the girl, will she recover? A. Yes, because the girl was engaged in an adult activity. B. Yes, because the girl did not behave as a reasonably prudent child of the same age, knowledge, intelligence, and experience under the circumstances. C. No, because her harm was caused by her boyfriend. D. No, because a 10-year-old child is presumed incapable of the state of mind necessary for negligence.

B. Yes, because the girl did not behave as a reasonably prudent child of the same age, knowledge, intelligence, and experience under the circumstances.

Alafair is, and knows she is, stunningly beautiful. At a singles' bar, she flirts with Bob, who is sitting beside her. Uninvited, she kisses him. He is horrified, upset and offended and sues Alafair for battery. Her testimony is that she meant no harm. A. Alafair should prevail because she caused no harm or offense. B. Alafair should prevail because a kiss is not the sort of touching that reasonable people view as harmful or offensive. C. Alafair should prevail in a dual intent state only, because while she intended to kiss him, she did not intend to harm or offend him. D. Alafair should prevail because she reasonably believed that a man in a singles' bar, flirting with a beautiful woman, would consent to a kiss. E. Alafair should prevail because any reasonable heterosexual man in a singles' bar would have consented to her kiss.

C. Alafair should prevail in a dual intent state only, because while she intended to kiss him, she did not intend to harm or offend him.

The principal area of concern of Tort law is: A. Preventing people or entities from acting in an anti-social manner. B. Enforcing norms or obligations the defendant voluntarily assumed. C.Requiring those who harm others (or their property) to compensate those they harm. D.Protecting an arena in which one is permitted to act largely without regard for the views of others. E.Setting out in advance detailed standards of behavior.

C.Requiring those who harm others (or their property) to compensate those they harm.

In a tort case, a corporate defendant that harms its neighbors in the course of business A. Will never be held liable for damages if it has obeyed the law. B. Should be punished proportionally to the danger of its actions. C. Poses a difficult problem for tort law because corporations do not obviously have consciences or mental states such as awareness of the difference between right and wrong. D. Reasonably might be viewed as having taken its neighbors' property or health and therefore ought to compensate them even if its actions were for a socially useful purpose.

D. Reasonably might be viewed as having taken its neighbors' property or health and therefore ought to compensate them even if its actions were for a socially useful purpose.

As applied to tort, the aphorism "you break it, you pay for it" emphasizes: A. Even if one is acting in an appropriate manner, if one uses resources (or other people's health and safety), one ought to pay for what one uses. B. In a capitalist, market-based, economy, prices are a critical way in which we allocate resources. For prices to be accurate, producers must pay for all the resources they use - even those that are not usually for sale, such as other people's health or the ecosphere. C. Tort liability is not punishment, even though it may feel that way to those who'd prefer not to pay to clean up their messes, just like requiring a grade-schooler to clean up after finger painting is not a punishment. D. All of the above

D. All of the above

As applied to tort, the aphorism "clean up your mess" emphasizes: A. Even if you do so innocently or in the course of a productive activity, if you hurt others or their property, you ought to take responsibility for your actions. B. One who injures others while acting within his rights is not responsible for the harm he causes. C. In a modern society, people are assumed to accept the risk of side-effects of beneficial activities by others, such as mining coal. D. The law follows the utilitarian principle of "the greatest good for the greatest number" even when that means that some individuals or their property must be injured.

A. Even if you do so innocently or in the course of a productive activity, if you hurt others or their property, you ought to take responsibility for your actions.

Domination By Design, Inc. a political action group advocating for free guns for all, publishes a political advertisement showing a cross-hairs centered around a picture of Poly -- a politician well-known for her gun-control work. The caption is "The tree of liberty must be watered from time to time with blood." Poly, aware that other politicians have been shot after similar ads, sues DBD in assault. A. DBD will prevail because any potential violence is not "imminent." B. DBD will prevail because Poly's fears are unreasonable. C. DBD will prevail because its advertisement is protected by the First Amendment. D. Poly will prevail because DBB intend to place her in fear of assassination.

A. DBD will prevail because any potential violence is not "imminent."

Defendant Dorald, an executive at Demonstrative Doubles, Inc. (DDI), had a discussion with several Muslim employees at the firm. The employees, Petra and Patish, explained that their religion forbids them to eat pork. Dorald, acting on behalf of DDI, agreed that the company's cafeteria, which regularly serves meatballs, would make the meatballs only from beef. The company did so. However, Dorald then decided that he had made a mistake and that permitting DDI employees to impose their religious beliefs on others was inappropriate. instead, the Muslim employees should behave like real Americans, eat pork and love it. To further the assimilation of Petra and Patish, he decided to change the meatball recipe to include pork, but not to announce the change. Some weeks later, Petra and Patish discover that they've been eating pork inadvertently. After violent fits of vomiting and much penitential prayer, they sue DDI for battery. Assuming that DDI is liable for Dorald's actions, which of the following is most likely correct? A. DDI is liable for battery because it intentionally touched plaintiffs in a manner that a reasonable person would view as offensive, by causing them to unknowingly eat pork in violation of religious strictures. B. DDI will prevail because it did not touch plaintiffs or force them to touch the meatballs. C. DDI will prevail because D's motive was to help Ps assimilate, which is an important social policy. D. DDI will prevail because the reasonable person likes pork meatballs.

A. DDI is liable for battery because it intentionally touched plaintiffs in a manner that a reasonable person would view as offensive, by causing them to unknowingly eat pork in violation of religious strictures.

On a motion for summary judgment, the uncontested evidence shows that Don believes that, like Midas, he can turn people into gold by touching them. He shakes hands with Patricia, gently and inoffensively, intending to turn her into gold. Of course, she doesn't turn into gold. However, Don's handshake inadvertently transmits a wasting disease to Patricia, who suffers horribly as a result. In Patricia's suit for damages in battery, the most persuasive argument is: A. Don is liable because he intended to harm her, and he did in fact harm her by touching her, even if the harm was not the harm he intended. B. Don is liable because he touched her in a manner that reasonable people would agree is harmful. C. Don is not liable because most people would agree that shaking hands is a socially appropriate action that is neither harmful nor offensive. D. Don is not liable because the injury he caused was entirely unintentional.

A. Don is liable because he intended to harm her, and he did in fact harm her by touching her, even if the harm was not the harm he intended.

Doris buys an antique ring from Frieda, who is not a merchant in the business of selling rings. Doris reasonably believes that the ring belongs to Frieda and she pays full market value for it. Unfortunately, it turns out that Frieda bought the ring from a thief (at full wholesale price and without knowing it was stolen). The thief stole it from the Museum of Contemporary Obsolescence. The Museum sues Doris in conversion. A. Doris is liable and must turn the ring over to the Museum because she exercises substantial dominion over the ring and the plaintiff has the right to possess it. B. Doris is liable for damages, but courts will not issue an injunction ordering her to turn over the ring. C. Doris is liable and must turn the ring over to the Museum, but the Museum must reimburse her for the price she paid to Frieda (and it has a good cause of action against Frieda and the thief to recover that amount). D. Doris is not liable because she is entirely innocent: she bought the ring in good faith for full value from a person who appeared to have the right to sell it. The Museum's remedies are only against those who knowingly or negligently wronged it.

A. Doris is liable and must turn the ring over to the Museum because she exercises substantial dominion over the ring and the plaintiff has the right to possess it.

Darrin is angry at Pedro. He decides to kill him. Pedro is standing on the street corner. Darrin is driving, about a block away. Spotting Pedro, Darrin accelerates his car to 80 mph aiming directly for Pedro. Fortunately, he narrowly misses hitting Pedro, zooming past within inches. Less fortunately, Pedro, startled by the near miss, has a heart attack and dies. Pedro's estate sues Darrin in tort for battery. Darrin, admitting the above facts, moves to dismiss. The court should: A. Grant the motion, because no reasonable jury could find that Darrin touched Pedro. B. Grant the motion, because attempted murder is a crime and crimes can only be prosecuted by the state. C. Grant the motion, because Darrin did not intend to cause Pedro to have a heart attack. D. Deny the motion, because Darrin intended to kill Pedro and by the doctrine of transferred intent, that intent is sufficient even though he killed him in a different manner than he planned.

A. Grant the motion, because no reasonable jury could find that Darrin touched Pedro.

A deaf bicyclist was riding on a trail commonly shared by bicyclists and pedestrians. The bicyclist hit and injured a pedestrian, who tried to warn the bicyclist that he was about to hit him by yelling, "Stop!" In a negligence action by the pedestrian against the bicyclist, to what standard of care will the bicyclist be held? A. He will be held to the same standard of care as a reasonably prudent deaf person. B. He will be held to the same standard of care as a reasonably prudent person. C. He will be held to the same standard of care as any person who is also deaf. D. There is not enough information from the facts to make this determination.

A. He will be held to the same standard of care as a reasonably prudent deaf person.

A real estate development company invited a man to tour a partially completed office building. The man entered the office building and began walking through the offices when a forklift operator negligently dropped a large lighting fixture nearby. The fixture broke and a piece of metal bounced off the ground, severely cutting the man on the leg. The man sued the real estate development company for his personal injuries. At trial, he established that a state statute required forklifts to be inspected by government agents. The man further proved that the company had failed to obtain such an inspection for that forklift, although that forklift was in fact in perfect condition when the accident occurred. Which of the following most accurately describes the legal effect of the company's violation of the equipment inspection statute? A. It has no bearing on the issue of the company's negligence. B. It conclusively proves that the forklift operator was negligent in dropping the lighting fixture. C. It establishes that the company was guilty of negligence per se regarding the man's injury. D. It renders the company strictly liable for the man's injury.

A. It has no bearing on the issue of the company's negligence.

A farmer owned a small farm near a town. The water to all outlets on the farmer's farm other than his house was supplied by a well drilled by a previous owner a few hundred feet behind the farmer's barn on the property. This well water became contaminated with arsenic. The congresswoman for that district who lived in the town was out hiking on the rural roads one very hot summer day when she got tired and decided to take a shortcut back to town over the farm fields. As she passed through the farmer's farm, she noticed a water faucet next to the barn and stopped to take a long drink of the cool water. The arsenic content of the water made the congresswoman immediately very sick. The farmer discovered her unconscious body a few hours later and rushed her to the local hospital. After a long period of painful convalescence, the congresswoman recovered with minor long-term damage to her health. The congresswoman brought an action for personal injuries against the farmer arising from her drinking of the arsenic-contaminated water. What will be the probable outcome? A. Judgment for the farmer, if the congresswoman was the first person ever to enter upon his farm without permission. B. Judgment for the farmer, unless the water faucet could be said to be an "attractive nuisance" under these circumstances. C. Judgment for the congresswoman, if the farmer knew that the well water was tainted with arsenic. D. Judgment for the congresswoman, if the farmer reasonably should have known that the well water was tainted with arsenic.

A. Judgment for the farmer, if the congresswoman was the first person ever to enter upon his farm without permission.

In the midst of a large storm, Homer notices that the rising flood waters seem likely to wash neighbor Ned's parked car into Homer's yard, which would cause serious damage to Homer's beloved and valuable rose garden and possibly endanger his house. To save the garden and to protect his home and his own physical safety, Homer opens the car door (which is unlocked) and releases the parking brake, causing the car to roll away from Homer's property and into a ditch. No one is hurt, the roses are safe, but the car is totaled. Ned sues Homer alleging conversion of chattels. A. Ned should recover because Homer was not privileged to damage Ned's property without paying for it. B. Homer is entitled to a sudden emergency instruction. C. Homer is not liable for conversion because he has an affirmative defense of necessity: his actions were justified to protect his own property. D. Homer is liable for conversion because he intentionally deprived Ned of his property.

A. Ned should recover because Homer was not privileged to damage Ned's property without paying for it.

The defendant and his cousin were driving home together from a nightclub. The defendant had not consumed any alcohol at the nightclub. On their way home, they observed a woman wandering haphazardly across an intersection. The cousin remarked, "I hope that girl sobers up. If she's doesn't pay attention, she could get killed." The defendant replied, "That's her own fault, and I have to get home. I'm not going to stop and help her." After being dropped off at his home, the cousin felt bad for the woman. The cousin asked his brother, who had not been drinking, to drive back to find and help the woman. When the brothers returned to the area where the woman had been, they found her lying in the middle of the street. They helped the woman into their car and drove her to the hospital. The woman had been struck by a car while she was wandering through the intersection. The car was never located. If the woman sues the defendant for damages for her injuries, will she prevail? A. No, because he was under no duty to help the woman. B. No, because the woman was involved in an abnormally dangerous activity. C. Yes, because a reasonable person would have helped the woman. D. Yes, because he breached his duty to the woman.

A. No, because he was under no duty to help the woman.

A vendor at a quarterly flea market sold everything from used clothing to packaged snacks. Her inventory depended upon the deals that the vendor came across during the quarter while hunting at garage sales and dollar stores. One time, the vendor purchased one hundred pre-packaged cupcakes that were deeply discounted for immediate sale. The vendor sold the cupcakes at the flea market the following weekend, even though it was a few days past the freshness date on the packaging. Several of the cupcakes were discovered to have glass fragments in the filling and a couple of consumers required stitches. Will the vendor be strictly liable under products liability for the injuries caused by the cupcakes she sold? A. No, because she is a one-time seller of the cupcakes. B. No, because she is not a retailer. C. Yes, because the products were sold past their freshness date. D. Yes, because commercial suppliers at all levels of the distribution chain may be held strictly liable.

A. No, because she is a one-time seller of the cupcakes.

A manufacturer of prefabricated homes received a shipment of drywall that was found to be defective in that it emitted a toxic vapor. The manufacturer returned the drywall but not before one home was completed using the defective drywall. The supervisors at the manufacturing company did not know that the defective drywall had been used and the home was sent out to a retailer with the defect. The retailer examined the home with the defect when it arrived on its lot and discovered that the home had been constructed with drywall that those in the industry had just learned was defective and posed a health risk. Aware of the concern, the retailer sold the home anyway. The couple who bought the home suffered from severe lung ailments and eventually traced their problem to the drywall in their new home. Is the manufacturer of the home liable under strict products liability? A. No, because the retailer discovered the defect before selling the home. B. No, because the retailer had the last clear chance to prevent the injury. C. Yes, because all commercial suppliers in the chain of distribution are strictly liable in products liability cases. D. Yes, because the defect existed when it left the manufacturer's control.

A. No, because the retailer discovered the defect before selling the home.

Doreen sees Pedro's child Paul acting very badly in the courtyard of the building where they all live, refusing to do his homework and smearing chocolate on another neighbor's dog. Doreen believes firmly in the Biblical aphorism, "spare the rod spoil the child." In order to save Paul from a lifetime of bad behavior, spoilation, misery and, undoubtedly, eventual crime, she slaps him. Shocked, Paul says, "what did you that for?" and when she explains, he sees that she was right. From then on, he behaves quite a bit better. However, he and his dad decide to sue Doreen in battery anyway. The above facts are undisputed. A. Paul should prevail if a reasonable person would find a slap by a non-parent to be offensive. B. Paul should prevail because he did not consent to being slapped. C. Doreen should prevail because, on balance, Paul suffered no harm. D. Doreen should prevail because Paul agreed after the fact that the slap was appropriate, thus consenting.

A. Paul should prevail if a reasonable person would find a slap by a non-parent to be offensive.

A biologist who lives in the city brought home a large dangerous reptile known as a komodo dragon. The lizard had attacked several people in the wild and had caused severe injury on at least two occasions. The biologist invited his friend over to see his pet lizard. The friend, who did not see the lizard in the middle of the hallway because it blended in with the color of the carpet, tripped over the lizard and broke his arm. The friend's arm became seriously infected and had to be amputated. The friend brought a suit in strict liability against the biologist. Who should prevail? A. The biologist, because the lizard did not attack his friend. B. The biologist, because the lizard was domesticated. C. The friend, because the biologist was strictly liable for injuries caused by the lizard. D. The friend, because the biologist did not exercise reasonable care with respect to the lizard.

A. The biologist, because the lizard did not attack his friend.

A businessman owns a delivery business in a small town, but his business is expanding. The businessman decides that he needs to lease a truck to satisfy the new demand for the delivery business. He contacts a car dealership and the two parties agree that the businessman will lease a truck made by a famous manufacturer. The businessman and the car dealership sign the contract and the businessman drives the truck out of the dealership. However, two weeks later, the businessman crashes while driving the truck because of faulty brakes. The businessman suffers serious injuries. Which of the following is correct? A. The car dealership and the manufacturer of the truck are strictly liable for the businessman's injuries. B. The car dealership is strictly liable for the businessman's injuries, but the manufacturer of the truck is not strictly liable. C. The car dealership is not strictly liable for the businessman's injuries, because the car dealership only leased the truck to the businessman. D. The car dealership is not strictly liable for the businessman's injuries, but the manufacturer of the truck is strictly liable.

A. The car dealership and the manufacturer of the truck are strictly liable for the businessman's injuries.

A carpenter was removing the shingles from her roof. She soon found that the container holding the old, newly removed shingles on the roof was full. Rather than get off the roof and empty the container, she simply slid the broken shingles off of the roof and let them smash onto the sidewalk. As she continued her work, she noticed a neighbor roller-skating close to the area that the singles were falling. The carpenter saw the neighbor begin skating on only one leg and with his eyes closed. The carpenter considered stopping her work until the neighbor was done skating, but chose not to do so. Suddenly, a few moments later, the neighbor skated on the sidewalk in front of the carpenter's roof at the same time that the carpenter slid a shingle down the side of the roof. The shingle slid down the roof and severed the neighbor's finger. The neighbor filed suit against the carpenter for negligence. The jurisdiction in question is a contributory negligence jurisdiction and applies the last clear chance doctrine. Which if the following is correct? A. The carpenter should be held liable for negligence, even though the neighbor was skating carelessly. B. The carpenter should be held liable for negligence unless the neighbor was skating carefully. C. The carpenter should not be held liable for negligence, because the neighbor was skating carelessly. D. The carpenter should not be held liable for negligence, because the neighbor would not have been injured but for his careless skating.

A. The carpenter should be held liable for negligence, even though the neighbor was skating carelessly.

After numerous traffic accidents at a certain intersection, a city decided to install a traffic signal there. The number of traffic accidents decreased dramatically. Six months later a concerned citizen, who lived near the intersection, noticed that the traffic light had stopped working. He immediately reported the outage to the city. Three days later, a driver came to a stop and then carefully proceeded through the intersection. However, he was hit by another car driven by a grandfather, who had failed to notice the signal was not working. The driver sued the city. Which of the following is the driver's strongest argument? A. The city failed to exercise due care by letting at least three days pass without repairing the signal. B. The driver was behaving reasonably when he entered the intersection since he stopped and then carefully proceeded. C. The combination of the city's failure to repair the signal and the grandfather's failure to notice the signal was out was the cause of the collision. D. A municipality is strictly liable for any injuries arising from its failure to properly control traffic flow.

A. The city failed to exercise due care by letting at least three days pass without repairing the signal.

Defendant Window Washing company was washing windows on a NYC high rise, 75 stories over the ground. Wendall, one of the washer's employees, was excitedly describing his spectacular new tool to his co-worker, when, waving his arms too vigorously, he dropped the tool. The company had not erected a sidewalk shed. 75 stories below, plaintiff Parrie Puck was seriously injured when the tool landed on his toe and bounced upward, breaking several critical bones. At trial plaintiff demands that the jury be instructed that persons using tools many stories above a busy street should exercise special care. Defendant objects to the instruction. A. The court should instruct the jury that it is the duty of every person to act with the care a reasonably prudent person would exercise under the circumstances. B. The court should instruct the jury that persons engaged in dangerous activities, such as washing windows high above the city, should exercise special care. C. The court should instruct the jury that the plaintiff may owe the defendant a special duty of care if they have a special relationship or if the defendant placed the plaintiff in peril. D. The court should instruct the jury that persons engaged in inherently dangerous activity are responsible for the damages they cause, regardless of how carefully they acted.

A. The court should instruct the jury that it is the duty of every person to act with the care a reasonably prudent person would exercise under the circumstances.

The defendant approached the plaintiff, who was standing at an ATM machine, and pressed a roll of quarters into his back and pretended to have a gun. The defendant then said to the plaintiff, "Give me $500 or I will shoot you with this gun." Believing the defendant would shoot him, the plaintiff complied with the demand. In a suit by the plaintiff against the defendant for assault, how will the court likely rule? A. The defendant is liable, because he committed an intentional act that caused the plaintiff to experience a reasonable apprehension of an immediate harmful contact. B. The defendant is liable, because he had the ability to inflict immediate harmful contact on the plaintiff. C. The defendant is not liable, because he did not actually have the ability to inflict immediate harmful contact on the plaintiff. D. The defendant is not liable, because he made a conditional threat to inflict immediate harmful contact on the plaintiff.

A. The defendant is liable, because he committed an intentional act that caused the plaintiff to experience a reasonable apprehension of an immediate harmful contact.

A man was jet skiing when he negligently caused his craft to sink. He remained floating in the water with the aid of his life preserver. The defendant, traveling alone in a boat which could hold four adults, saw the man in the water. He went over to the man and stayed long enough to determine that the man was not injured. However, the defendant was running late for work and figured that someone else would come by soon. He also told the man that he would alert the coast guard once he got inland. The defendant then sped away without helping the man into his boat. Two hours later, the defendant reached land and notified the authorities. When the coast guard found the man, he was severely bleeding after having been crushed by a woman's huge yacht. The woman had seen the man in time to avoid him, but was so drunk that she could not steer the yacht aside fast enough. The man sued the defendant for his injuries. For whom will the court grant judgment? A. The defendant, because he did not leave the man in greater peril than he found him. B. The defendant, because the man was responsible for his own predicament due to his own negligence. C. The man, because the defendant incurred a duty to rescue him by stopping and investigating his situation. D. The man, if a reasonable person would have picked him up under the same circumstances.

A. The defendant, because he did not leave the man in greater peril than he found him.

The plaintiff was visiting a friend when the friend's pet dog walked into the room. The plaintiff was frightened of the dog and tried to leave the room, but the dog was blocking the only doorway out of the room. The plaintiff then tried to distract the dog so he can leave the room but only succeeded in startling the dog, who bit him. Which of the following arguments would best support the plaintiff's suit against his friend under a strict liability theory? A. The friend knew that the dog bit someone last week. B. The dog is generally held to have known dangerous propensities. C. The dog growled at the plaintiff before biting him. D. The dog's breed is known for being aggressive.

A. The friend knew that the dog bit someone last week.

A man was hired as a security guard by a movie production company. The movie set was in a large warehouse in a rural area. During the security guard's first day on the job, the supervisor told the new guard that company policy required him protect himself if necessary and to call the police at the first sign of trouble. The guard was told to stay on the leased premises when on duty. One night, a group of teenagers walked past the set. The security guard became suspicious so he followed them. When the teenagers noticed the guard, they began to throw pebbles at him. After a few minutes, the guard bolted towards the teenagers. He chased them down, grabbed the tallest of the group, handcuffed him and punched him in the face. The injured teenager filed a lawsuit against the security guard and the production company. Which of the following is the best argument in the production company's defense? A. The guard was not acting within the scope of his employment. B. The guard was defending the company's property. C. The guard was acting in self-defense. D. The guard was contributorily negligent.

A. The guard was not acting within the scope of his employment.

A pool owner purchased a new robotic pool cleaner. A week later, the owner received a severe bruise when the robot attached to his arm when he was swimming. The owner realized that there was a problem with a sensor that failed to shut off power when the robot attached to a person. The owner called the store that sold the pool cleaner and arranged to return it for a full refund. The owner was busy, however, and did not have time to make a trip to the store for several days; so he put the pool cleaner in a corner in his pool shed to store it until he had a chance to return it to the store. The next day, the pool owner had a pool party. The owner had a big pool side barbeque and loved to have pool guests. Shortly after the guests arrived, however, the pool owner had a work emergency and had to leave the pool for a few hours. He told all the guests to make themselves comfortable while he went to the office. While the pool owner was away from the pool, a guest spilled a bag of chips into the pool. Wanting to be a polite guest, she decided to run the pool cleaner before the pool owner returned. She saw the new robot cleaner in the corner of the shed, put it in the pool and pressed the start button. The robot latched on to the guest and caused a large bruise on her leg. The injured guest asserted a claim against the pool owner. Which of the following is the most likely result? A. The guest will recover, because the pool owner knew about the hazardous condition of the pool cleaner and failed to warn the guest. B. The guest will recover, because the pool owner had a duty to make a reasonable inspection of the pool to discover any unknown dangers. C. The guest will not recover, because the pool owner did not create the risk of harm. D. The guest will not recover, because of the family purpose doctrine.

A. The guest will recover, because the pool owner knew about the hazardous condition of the pool cleaner and failed to warn the guest.

One afternoon, a homeowner was working in his backyard when he noticed that a satellite television antenna on his property was hanging over his back fence and was positioned high above the sidewalk that ran along the street. The homeowner thought the structure looked weak and about to break. He contacted an antenna installation and repair service to have them evaluate and repair the antenna. The repair company employee assigned to the job climbed on the roof and started to repair the structure. The employee negligently failed to properly mix the structural adhesive used for such repairs. A week later, the antenna fell onto the sidewalk and injured a man who was riding his bicycle by at the time. The man filed suit against the homeowner. Which is the most likely reason that the man prevailed? A. The homeowner is liable for the maintenance company's negligence as this is a non-delegable duty. B. The homeowner is liable on the basis of respondeat superior. C. The homeowner strictly liable, because the dangerous antenna structure was on his property. D. The homeowner was liable, because all tortfeasors are jointly and severally liable for the injuries caused.

A. The homeowner is liable for the maintenance company's negligence as this is a non-delegable duty.

One night, a boxer was sitting in a pub enjoying a beer when a man sat next to him at the bar. The man had just had a fight with his wife and was in a foul mood. After a few minutes, the boxer's cell phone rang and the boxer answered the call. The man was incensed that someone would use their cell phone in a public establishment. He started screaming at the boxer to get off the phone. The boxer tried to move down the bar away from the man, but the man followed him continuing to yell. The bartender witnessed the events but chose not to get involved. The boxer was finally so aggravated by the man's yelling that he turned and punched the man in the face. What is the likely result if the man sues the pub for battery? A. The man will not prevail because the boxer was not an employee of the pub. B. The man will not prevail because he assumed the risk. C. The man will prevail if the bartender was negligent in not intervening. D. The man will prevail if the offensive contact was intentionally inflicted.

A. The man will not prevail because the boxer was not an employee of the pub.

A yacht owner hired a captain and a mate to sail his boat from New York to Miami. On the first day at sea, the captain climbed to the top of a 32-foot mast checking for structural cracks. Not paying attention to what he was doing, the mate grabbed a ladder out from under the captain's feet without looking up to see whether the captain was using it. The captain screamed for help and hung on to the mast for his life. The yacht owner, who was playing ping-pong on deck with his friend, had been drinking rum and cola all day. Hearing the captain's cry for help, the yacht owner looked up to see the captain hanging onto the mast high above the deck. The yacht owner told the captain to hold on. The yacht owner hatched a foolish plan to have the captain jump into a small fishing net being held by the captain's drunken friend. The rescue plan would never work because the yacht owner overestimated the strength of the net. The mate almost had the ladder back under the captain's feet when the yacht owner climbed up the mast and released the captain's hands so that the captain would drop safely into the net below. The captain fell through the net and broke both legs. Which of the following is correct? A. The mate should be held liable for negligence, because she breached her duty of care. B. The mate should be held liable for negligence, because her act constituted negligence per se. C. The mate should not be held liable for negligence, because the yacht owner's rescue attempt was unforeseeable. D. The mate should not be held liable for negligence, because the yacht owner's negligence was an intervening force.

A. The mate should be held liable for negligence, because she breached her duty of care.

A 13-year-old girl used a skateboard path along a busy road to get to her friend's house. The girl's parents knew that she regularly used the path to skateboard to her friend's house. One early spring day it began to hail. A driver on the road negligently skidded into the girl, killing her instantly. The girl's parents bring a suit against the driver for wrongful death. The jurisdiction has adopted the rule of contributory negligence. Under which of the following circumstances will the girl's parents be entitled to recover? A. The parents can recover damages only if neither they nor the girl was negligent. B. The parents can recover damages even if both they and the girl were negligent. C. The parents can recover damages even if the girl was negligent, as long as they were not negligent. D. The parents can recover damages even if they were negligent, as long as the girl was not negligent.

A. The parents can recover damages only if neither they nor the girl was negligent.

A "defendant" is: A. The person being sued. B. The person suing. C. A person who has lost a lawsuit. D. A person who has been injured. E. A person who injures another.

A. The person being sued.

The defendant owns and operates a small store in which he repairs used appliances and resells them to the public. Recently the defendant repaired a toaster and sold it to the plaintiff. When the plaintiff was using the toaster, it malfunctioned and burned his hand. The toaster malfunctioned because the defendant had not repaired it correctly. The plaintiff brought suit against the defendant for the injuries arising from his use of the toaster. Which of the following is correct? A. The plaintiff will prevail in his suit against the defendant because the defendant was negligent in repairing the toaster. B. The plaintiff will prevail in his suit against the defendant because the defendant is strictly liable for any injuries caused by the malfunctioning toaster. C. The plaintiff will not prevail in his suit against the defendant because the defendant was not negligent in repairing the toaster. D. The plaintiff will not prevail in his suit against the defendant because the defendant was not the original manufacturer of the toaster.

A. The plaintiff will prevail in his suit against the defendant because the defendant was negligent in repairing the toaster.

`The defendant is eight years old, and she is very mature and intelligent. After observing her parents operate the family car a number of times, the defendant decides to drive the car down the block to visit a friend. While driving the car, the defendant hits and injures the plaintiff, who is walking on the side of the road. The plaintiff brings a suit for negligence against the defendant. Under the majority rule, to what standard of care will the defendant be held? A. The standard of care of a reasonably prudent adult. B. The standard of care of a child of similar age. C. The standard of care of a child of similar age, intelligence, education and experience. D. Because the defendant is between the ages of seven and 13, it will be rebuttably presumed that defendant was not negligent.

A. The standard of care of a reasonably prudent adult.

A young woman enters a clothing store to buy a new outfit for college. After trying on a few outfits, she finds just what she was looking for. At the checkout counter, the clerk stared at the woman and made her feel uncomfortable. She asked the clerk for her credit card back and he informed her that it is a stolen card. The clerk grabbed the woman's purse off the counter and said "Now you can't leave until the cops get here". The woman feels she has no choice but to stay because her purse contains her car keys, wallet, and phone. The woman filed suit against the store for false imprisonment. Who should prevail? A. The woman, because she was forced to stay against her will. B. The woman, because there was no reasonably apparent means of escape. C. The store, because the clerk intended to prevent a crime. D. The store, because the woman was free to leave.

A. The woman, because she was forced to stay against her will.

A college student purchased a small computer that was designed for both portability and for desktop use. The computer had short legs in the back that pulled out and raised the bottom of the computer up at a slight angle. The student assumed that the legs were there to make the keyboard more ergonomic. When one of the legs would not pull out evenly with the other, the student simply pushed both legs back in and used the computer with its bottom flat to her desk. While she was out of her room, the battery case on the computer dangerously overheated because it could not vent as designed to with the legs in place. It caused a fire that burned all of the contents in two adjoining dorm rooms before it was put out. Luckily, nobody was injured. Later it was determined that the student's particular computer had a defect that caused one of the legs to stick. Is the manufacturer of the computer liable under strict products liability in a jurisdiction that has not adopted a comparative negligence system for products liability? A. Yes, because the computer's defect caused the damage. B. Yes, because the computer's defect was not detected by the student. C. No, because there were no personal injuries. D. No, because the student perceived the defect in the computer.

A. Yes, because the computer's defect caused the damage.

The plaintiff suffers from brittle bone disease. Because he is not paying attention to where he is going, the defendant bumps into the plaintiff while walking down the street. As a result, the plaintiff falls and fractures several bones, requiring hospitalization. If the plaintiff did not suffer from brittle bone disease, she would not have fractured any bones. Is the defendant liable to the plaintiff for her injuries under a theory of negligence? A. Yes, because the defendant will be liable for the plaintiff's injuries even if the consequences are more severe than they would have been in a person not suffering from brittle bone disease. B. Yes, because the defendant should have known that the plaintiff had from a condition that would cause her to suffer more severe injuries than a person not suffering from brittle bone disease. C. No, because a defendant will not be liable for the plaintiff's injuries if the consequences are more severe than they would have been in a person not suffering from brittle bone disease. D. No, because the defendant did not have knowledge that the plaintiff had from a condition that would cause her to suffer more severe injuries than a person not suffering from brittle bone disease.

A. Yes, because the defendant will be liable for the plaintiff's injuries even if the consequences are more severe than they would have been in a person not suffering from brittle bone disease.

A father regularly observed his 13-year-old son taunting the eight-year-old neighbor. On several occasions, the father overheard his son threatening to "beat up" the neighbor, but the father never took action to intervene in his son's behavior, figuring "boys will be boys." Recently, the son hit the neighbor with a baseball bat, causing serious injury to the neighbor. Will the father likely be liable for the son's actions? A. Yes, because the father had the ability to exercise control over his child and failed to do so. B. Yes, because a parent always has a duty to exercise reasonable care to control his minor child. C. No, because a parent never has a duty to exercise reasonable care to control his minor child. D. No, because the father did not know that the son was going to attack the neighbor.

A. Yes, because the father had the ability to exercise control over his child and failed to do so.

A salesman was driving to work in a crowded downtown area. A brick thrown from the third story window of a warehouse struck the salesman's car and caused him to swerve into a crowd of pedestrians. Several of the pedestrians were hospitalized with broken bones. The company that leased the warehouse employed eight people at the time of the incident. None of employees admitted to throwing the brick. Two of the injured pedestrians asserted a negligence claim against the owner of the company that leased the warehouse. The defendant has admitted that no one other than his employees were present in the building at the time of the incident. Will the pedestrians be successful? A. Yes, under the doctrine of res ipsa loquitur because the brick was within the control of the company. B. Yes, because a brick is not usually thrown from a window in the absence of someone's negligence. C. No, because the brick was not within the control of the company at the time the pedestrians were injured. D. No, because the company is not vicariously liable for the intentional torts of its employees.

A. Yes, under the doctrine of res ipsa loquitur because the brick was within the control of the company.

A man is aware that a woman walks across the southeastern corner of the man's property, without the man's permission, each afternoon on her way home from work. The man has recently learned that one of the large oak trees in the southeastern corner of his land has rotted and will probably fall in the very near future. The condition of the tree is not readily apparent merely from looking at it. What duty, if any, does the man owe to the woman with regard to the oak tree in a majority of jurisdictions? A. The man owes no duty to the woman because she is a trespasser. B. The man owes no duty to the woman because the rotten oak tree is a natural condition on the land. C. The man must warn the woman of the hidden danger caused by the oak tree. D. The man must exercise reasonable care to protect the woman from injuries that may be caused by the oak tree.

B. The man owes no duty to the woman because the rotten oak tree is a natural condition on the land.

A manufacturing company sold logging equipment for the general public. The company did not place warnings on its saws, axes, and other tools. A grocer, who was not a professional logger, bought one of the company's axes from his local hardware store. While trying to cut some logs with his new axe, he grabbed the axe incorrectly, and hurt himself. The grocer sued the manufacturer. Which of the following is correct? A. The grocer prevails, because he was not a professional logger. B. The grocer prevails, because the axe required a warning. C. The company prevails, because it did not sell the axe to the grocer. D. The company prevails, because the axe did not require a warning.

D. The company prevails, because the axe did not require a warning.

A woman was injured when she used a curling iron on her hair, which, when on the highest setting, caught fire and burned the woman's hair, scalp, and face. It appears that the defect was caused when the curling iron was manufactured. The curling iron was manufactured by a manufacturer, distributed to a retailer by a distributor, and sold in a store by the retailer. The retailer asked a local hairdresser to put a display of the curling irons in his salon to see if customers liked them. The woman bought the curling iron from the hairdresser when she saw the display in his salon after a hair appointment. The woman now seeks to bring a strict products liability action against all possible parties for her injuries. Against whom may the woman properly bring suit? A. The manufacturer, the distributor, the retailer, and the hairdresser. B. The manufacturer, the distributor, and the retailer only. C. The manufacturer only. D. The manufacturer, the distributor, and the hairdresser only.

B. The manufacturer, the distributor, and the retailer only.

David is an affluent professional who recently moved into a rapidly gentrifying neighborhood. Unfortunately, he found that the local addicts have been using his entrance lobby as a convenient indoor space to smoke crack. Repeated calls to the police didn't help. His new locks didn't help either: the addicts simply cut them out. Neither did a massive mahogany door; within a week of its installation, someone had broken it as well. Tired of repeated repairs, not to mention terrified for his and his girlfriend's physical safety should they surprise someone in the act, he decided that he needed to take more drastic measures. He posted a sign on the door with a skull and crossbones and the words: "Absolutely No Trespassing. Trespassers Risk Severe Physical Injury or Death. Do NOT Enter For Any Reason. Authorized or Invited Individuals should call the following phone number for admission." A second sign said "No Smoking - Extreme Danger." He then rigged up an ultra-sensitive smoke detector in the lobby; instead of sounding an alarm, the detector served as a switch to release a painful gas into the lobby. Penny, a neighbor, saw David's door ajar one day and went in, smoking a cigarette, to offer this obviously needy person the Good News that salvation is near. She was seriously injured by the poison gas and sues for battery. A. David is liable for battery because gas is sufficiently material to constitute a touching. B. David is liable for battery because his defense is not proportional to the threat posed. C. David is not liable for battery because he has an inherent right of self-defense when the police are unable or unwilling to protect him. D. David is not liable because Penny ignored his clear warnings.

B. David is liable for battery because his defense is not proportional to the threat posed.

Dimni read on the internet that a book can stop a bullet. He experimented, shooting at a book in his backyard; the bullet seemed to be stopped. Seeking a more accurate test, he went to the nearest bookstore. After identifying a patron holding a book in front of her chest, Dimni walked close to her, pulled out his gun and shot the book. Unfortunately, it turned out that the internet isn't always reliable, and he hadn't taken into account the difference between shooting from the opposite side of the yard and shooting at point blank range. The patron, Patty, was seriously injured. Patty sues Dimni in battery. Dimni testifies, and Patty concedes, that his sole motive was the advancement of science and he wished no harm on Patty or anyone else. Which of the following is most accurate? A. Defendant should prevail in a dual intent state because he had no intent to harm. B. Defendant should be found liable because he intentionally fired his gun, causing an injury that is obviously harmful. C. Defendant should prevail because he did not intend to touch Patty. D. Defendant should be held liable if, and only if, Patty can establish by a preponderance of the evidence that Dimni was aware that gunshots at close proximity ordinarily cause injury.

B. Defendant should be found liable because he intentionally fired his gun, causing an injury that is obviously harmful.

Donald commissioned a survey of his urban lot from a reputable survey firm. Unfortunately, the surveyor erroneously marked the eastern boundary of the lot one foot to the east of the "true" line. Donald then proceeded to build a seventy story tower. Because of the surveyor's error, the eastern wall of the building is actually on Petronius's land. Fixing the problem would require dismantling the entire building and rebuilding it, at enormous expense. Petronius uses his land as a storage and collection yard for garbage from other buildings in the area; the loss of a foot does not materially affect his use. Petronius sues Donald for trespass. Under standard tort doctrine, the court should hold: A. Donald is liable for trespass and Petronius should be awarded the market value of the land that Donald is using. B. Donald is liable for trespass and Petronius should be awarded an injunction ordering Donald to rebuild the building entirely on Donald's land. 3. Donald is not liable for trespass because the enormous cost of correcting the problem trumps Petronius's rights. 4. Donald is not liable for trespass because he did not intend to invade another's land.

B. Donald is liable for trespass and Petronius should be awarded an injunction ordering Donald to rebuild the building entirely on Donald's land.

Giant Utility Co. burns coal to power its electric plant. It concedes that it has long known that burning coal releases sulphur into the air, that natural processes convert the sulphur into sulphuric acid, and the sulphuric acid is carried by the prevailing winds long distances before eventually precipitating out in acid rain. It also concedes that acid rain causes various forms of personal injury and property damage. General Utility also acknowledges that electricity can easily be produced by methods that produce different or fewer harms, but when the plants were built coal was the cheapest method. General Utility correctly contends that were it to convert its coal plants to alternatives or abandon them, it would incur additional costs (which, in turn, would have to be paid by investors in the form of lower profits or consumers in the form of higher prices). A group of plaintiffs who have suffered both personal injury (damaged lungs) and property damage (damaged exteriors to their buildings) sue Giant Utility Co, asserting claims in battery, trespass and conversion. Which of the following is the most likely result? A. Giant Utility is liable because it is deemed to have intended the inevitable consequences of its actions, and it was aware that injury was inevitably the result of its coal burning. B. Giant Utility is not liable for an intentional tort, because it created a risk of injury, and creating a risk -- even intentionally creating a risk that, statistically, is virtually certain to harm someone eventually -- does not meet the intent or act requirements for battery, trespass or conversion. C. Giant Utility is liable in battery because it intentionally created physical particles and gases that it knew would touch other people's bodies and invade their property without their consent, and that harm would result, and harm did result. D. Giant Utility is not liable because it produces electricity, an essential component of modern, civilized, life. If injuries are the necessary consequence, they are a small price to pay for civilization, and part of the cost we must all assume for living with modern conveniences.

B. Giant Utility is not liable for an intentional tort, because it created a risk of injury, and creating a risk -- even intentionally creating a risk that, statistically, is virtually certain to harm someone eventually -- does not meet the intent or act requirements for battery, trespass or conversion.

In the last legislative session, the legislature enacted a statute permitting factories to operate boilers at a higher pressure than previously was legal. The old standard was 250 psf; the new one is 500 psf. The local factory owners group lobbied heavily for the new statute, arguing that the higher pressures would allow factories to produce more goods at a lower cost, thus leading to more rapid economic growth and increased jobs, and that safety concerns were overstated. Opponents argued that the higher pressures were unsafe, but they were defeated. The Dimdom Corporation operates a factory. Three years ago, it increased the pressure in its factory boilers to 500 psf - illegal then, but now permitted. It has had two major accidents, one just before the change in the statute and one right after. The accident victims sue in negligence. In each case, the parties agree that the accident would not have happened had the pressure been at 250 psf. Dimdom, however, asserts that its decision to operate at the higher pressure was reasonable in light of the safety record of the boilers and the benefits of operating at 500 psf, and, moreover that its decision was vindicated by the legislative enactment. Based on the uncontested causation evidence and the statutes, each side moves for summary judgment. The court should: A. Grant summary judgment to plaintiffs on the first count on the ground that the statute sets the applicable standard of care, so that defendant's illegally high pressure was negligence per se. Grant summary judgment to defendant on the second count on the same ground: the new statute sets the new standard of care. B. Grant summary judgment to plaintiffs on the first count on a theory of negligence per se. Deny summary judgment on the second count. After trial, instruct the jury that statutory compliance may be evidence of reasonable behavior, but the jury must make an independent assessment of whether the defendant acted with the care that a reasonably prudent person would exercise under the circumstances. C. Deny summary judgment on both counts and instruct the jury that while the statutory standards may be evidence of reasonable behavior, the jury must make an independent assessment of whether the defendant acted with the care of a reasonably prudent person. D. Grant summary judgment to defendant on both counts on the ground that the legislature's recent enactment sets the standard of care, and the defendant complied with that standard even when it was in violation of the obsolete statute.

B. Grant summary judgment to plaintiffs on the first count on a theory of negligence per se. Deny summary judgment on the second count. After trial, instruct the jury that statutory compliance may be evidence of reasonable behavior, but the jury must make an independent assessment of whether the defendant acted with the care that a reasonably prudent person would exercise under the circumstances.

The plaintiff swallowed a small piece of glass while drinking a bottle of cola that was manufactured by the defendant. As a result, the plaintiff suffered several injuries, including a cut to her esophagus. The glass in the cola bottle will be classified as what type of defect? A. Absence of warning. B. Manufacturing defect. C. Design defect. D. Negligent defect.

B. Manufacturing defect.

A statute in State X provides that: "It shall be unlawful for any person to sell, or permit to be sold, intoxicating liquors to one who visibly is under the influence of alcohol. Whosoever shall sell intoxicating liquors in contravention of the foregoing provision shall be guilty of a misdemeanor punishable by imprisonment for not more than 60 days or a fine of not more than $5,000 or both." The owner of a local bar and grill brought the statute to the attention of his bartenders. Late one night, a patron entered the bar and ordered a gin martini. Noticing that the patron was very drunk, the bartender said, "Hey buddy, I don't think you can handle another drink." The patron responded, "C'mon, just one more for the road." The bartender acquiesced and mixed a martini from a bottle of gin, which he then served to the patron. The bartender was unaware that a disgruntled employee of the gin distiller had spiked the bottle with strychnine, a highly poisonous substance. After drinking the martini, the patron left the bar and began to drive home. Moments later, due to the poison he became violently ill and went into convulsions. As a consequence, the patron lost control of his car, which struck and seriously injured a pedestrian. The patron, who subsequently recovered, was prosecuted under a state statute making it a misdemeanor to drive while under the influence of alcohol. If the pedestrian asserts a claim against the bartender, will the bartender likely be held liable? A. No, because the pedestrian was not a foreseeable plaintiff. B. No, because the bartender's act was not the legal cause of the accident. C. Yes, because he was negligent per se in serving the patron the martini. D. Yes, because but for the fact that he served the patron the martini, the pedestrian would not have been injured.

B. No, because the bartender's act was not the legal cause of the accident.

A manufacturer of residential cleaning supplies introduced a line of liquid dish detergents in various colors that reflected the fragrances that the products already had. For example, a product with a citrus fragrance was colored bright orange. The containers had printed on them all the usual safety warnings that the manufacturer put on its original line of detergents, including a caution to keep the product out of the reach of small children. The manufacturer was not required by law to put child-safety caps on the detergents. A small child fell ill, briefly, after drinking several gulps of the orange colored citrus detergent and the child's parents sued the manufacturer for having colored the detergents in such a way that they resembled fruit juice. Will the court apply strict products liability to the manufacturer for having made its product more attractive to small children? A. No, because children are not the intended users. B. No, because the containers carried safety warnings. C. Yes, because the same safety warnings as before were not adequate once the colors of the detergents became more attractive to children. D. Yes, because the product contained a design defect.

B. No, because the containers carried safety warnings.

A woman owned a pet tiger cub, despite the "No Pet" rule in her apartment building. Most of the time, she kept the cub locked in her guestroom, which had no windows, so neighbors would not see the cub from the street. However, she occasionally let the cub loose to play around her apartment, so that he wouldn't get depressed by being confined to one room. The cub generally acted like a house cat, and when he was allowed out of the guestroom, he spent most of his time sleeping on a corner of the couch. The cub was very friendly, but unaware of his strength and sometimes bit too aggressively when playing with chew toys. One evening, the woman invited her friend over to watch a movie. The friend was aware that the woman had a pet tiger cub, but he did not know that she let it out of the guestroom occasionally. When the friend arrived, the woman forgot to mention that the cub was sleeping on the couch. While the woman was in the kitchen making popcorn, the cub woke up and wanted to play with the friend. The cub grabbed its favorite chew toy, carried it over to the friend and dropped it at his feet. Annoyed, the friend started batting the cub across the face with the toy. The cub snapped at the friend, biting his finger. The friend jumped backward and fell over the edge of the couch, breaking his arm. If the friend sues the woman for his injuries, should he prevail? A. No, unless the woman knew that the tiger cub had dangerous propensities. B. No, because the friend knowingly and unreasonably subjected himself to a risk of harm. C. Yes, because, as owner of the tiger cub, the woman is strictly liable for the friend's injuries. D. Yes, because the woman was negligent in failing to warn the friend that the tiger cub was loose.

B. No, because the friend knowingly and unreasonably subjected himself to a risk of harm.

A pharmaceutical company produces various medical treatments and prescription drugs. One such drug is intended to cure insomnia. Before the pill is released to the general public, tests show that it produces two side effects. One such side effect is dizziness that occurs in five of every one million people that take the drug. The other side effect is a serious illness that occurs in two of every one million that take the drug. Since the side effects are so rare, the company decides not to include warnings about the side effects on its product. When the drug is released into the market, two people file strict product liability actions against the company. One is a man who took the drug and experienced dizziness as a result. The other is a woman who took the drug and became gravely ill as a result. Which of the following is correct? A. Only the man will prevail, because the company did not include a warning about dizziness on its product. B. Only the woman will prevail, because the company did not include a warning about serious illness on its product. C. Both the man and the woman will prevail, because the company did not include warnings about either dizziness or serious illness on its product. D. The company will prevail against both the man and the woman, because the company didn't need to place either warning.

B. Only the woman will prevail, because the company did not include a warning about serious illness on its product.

Donald is deluded and believes he has the powers of Midas -- to turn anything he touches into gold. With malice in his heart, he heads to the home of his worst enemy, Peter, intending to turn Peter into gold. He rings Peter's doorbell and says, "Prepare to meet the Goldsmith. Bwa ha ha ha ha!" Peter is terrified -- Donald seems quite deranged and capable of anything. Donald then touches Peter and says "So long sucker." He turns and leaves, imagining that he has turned Peter into a statue. Peter is stupefied, but not for long; he calls his lawyer and tells him to sue. As Peter's lawyer, what advice do you give him? A. Peter does not have a claim in assault because Donald intended to touch him -- and did touch him. B. Peter probably has a successful action for assault, but only if he can establish that his terror was based on a reasonable belief that Donald was about to do him bodily harm. C. Peter does not have a claim in assault because Donald's intentions were irrational. D. Peter probably has a claim for assault because he was terrified as a result of Donald's intentional actions.

B. Peter probably has a successful action for assault, but only if he can establish that his terror was based on a reasonable belief that Donald was about to do him bodily harm.

Den owns and operates a small grocery store in downtown Hempstead. One day a group of rowdy teenagers entered his store together and began calling him names. One of them - Pierre - grabbed a piece of candy and waved it as if he was going to take it, while shouting "I'm Pierre and I don't care." Den alerted his security guard, who grabbed the kid, twisted his arm and forced him into a corner, where he kept him cowering for several hours while threatening to call the police and have him sent to prison upstate for a decade. The other teens ran. Den, to be sure they got the message, fired a gun over their heads. Pierre sues for assault, battery and false imprisonment. Common law as set out in Restatement 2d applies. A. Pierre should prevail because the shopkeeper's privilege does not apply where no shoplifting has actually taken place. B. Pierre should prevail because Den's actions amounted to false imprisonment, battery and (probably) assault and were not designed to investigate the facts. C. Den should prevail because he used reasonable force to recapture a chattel while in hot pursuit. D. Den should prevail if he can establish that he reasonably believed that Pierre was committing a crime.

B. Pierre should prevail because Den's actions amounted to false imprisonment, battery and (probably) assault and were not designed to investigate the facts.

The great change in the law symbolized by Brown v. Kendall is: A. Prior to Brown v. Kendall, the common law ordinarily did not find liability for merely acting negligently, i.e., without due care. B. Prior to Brown v. Kendall, the common law imposed liability without regard to care when an actor directly injured another. C. Prior to Brown v. Kendall, the common law imposed liability on people who hurt others while beating dogs. D. Prior to Brown v. Kendall, the common law required those who cause damage to others to compensate their victims.

B. Prior to Brown v. Kendall, the common law imposed liability without regard to care when an actor directly injured another.

Defendant Wendell Window Washing Company is in the business of washing windows on the exterior of NYC high rises. Periodically, it has accidents -- things drop, platforms fall, winches fail, wind blows. Recently, a new electronic device has become available that monitors each of the moving parts of the platform and its safety devices, sending a message to headquarters if any part needs maintenance or adjustment. The Monitor is not cheap; using it would require the company to increase its prices 5%, making them more expensive than leading competitors. Management believes that many customers would switch providers were WWWC 5% more expensive than its competitors. Accordingly, it concludes that it would be unreasonable to use the Monitor, even though it likely would reduce the number of accidents. After an accident, plaintiff asserts that the accident would have been avoided had the WWWC used the Monitor. Defendant moves for summary judgment dismissing the case on the ground that on the undisputed facts, it had no duty to use the device. A. The court should deny the motion because the duty of care ordinarily is a jury question. B. The court should deny the motion because the defendant has not provided sufficient evidence to determine whether or not it was reasonable to omit the Monitor. C. The court should grant the motion because the duty of care does not require defendant to take all available care, but only reasonable care, and here the defendant conformed to the industry standard. D. The court should grant the motion because no jury could rationally conclude that a reasonable company would use a device that would put it out of business.

B. The court should deny the motion because the defendant has not provided sufficient evidence to determine whether or not it was reasonable to omit the Monitor.

An executive has been living with her family in a house for three months when her company transfers her to another city. She hires a broker, and the broker sells the house to a young businessman. Shortly after the young businessman moves in, he is hurt by a piece of the ceiling that falls on his head. The businessman sues the executive and the broker for his injury. Which of the following is correct? A. The executive will be strictly liable, because she is in the chain of distribution. B. The executive will not be strictly liable, because she did not construct the house. C. The broker will be strictly liable, because she is in the chain of distribution. D. The broker will not be strictly liable, because she is a service provider.

B. The executive will not be strictly liable, because she did not construct the house.

In an effort to boost business, a local theme park installed a 70-foot-high roller coaster called "The Serpent." Because it was the only monster coaster for hundreds of miles, the owners hoped kids would bring their parents to the park. The park hired a highly specialized independent contractor to install, operate, and maintain the coaster. The plan worked beyond anyone's expectations, and business in the park doubled. One day, a ten-year-old girl came to the park with her mother. After a long day of fun, they lined up for the coaster. As the girl was boarding, her hand became wedged in the safety bar. An employee of the specialized independent contractor had neglected to replace and test the micro-switch that released the bar in case of interference or resistance. As a result, the girl's hand was caught in the moving gears. A park employee was walking by the girl when the girl's hand was caught and was the closest person to the scene. Had the employee hit the safety release quickly enough the girl would only have suffered superficial cuts and bruises. Instead, because the employee failed to act immediately, the girl's hand was mangled and required three reconstructive surgeries. The girl's mother filed suit against the theme park on behalf of the girl. Who should prevail? A. The girl, because the park employee failed to act as a reasonable person. B. The girl, because an employee of the contractor was negligent in not replacing and testing the micro-switch. C. The park, if the girl was also at fault. D. The park, because the girl's mother assumed the risk by letting her ride on "The Serpent."

B. The girl, because an employee of the contractor was negligent in not replacing and testing the micro-switch.

A fisherman went out fishing in a small boat despite the fact that the weather forecast included a small craft advisory. The fisherman, who was not wearing a life preserver, was thrown into the water when a wave hit his boat broadside. A passing jet skier saw the fisherman fall in the water, but he was late for dinner so he decided to head for shore before calling for help. The jet skier figured that, at worst, the fisherman would have to swim for a few minutes until help arrived. Unfortunately, the fisherman had a leg cramp and could not stay afloat. By the time the jet skier called 911, and the coast guard dispatched a rescue helicopter, the fisherman had drowned. The fisherman's wife filed suit against the jet skier for damages. The jet skier filed a motion to dismiss the complaint. Which if the following is the best argument in support of the motion to dismiss? A. The jet skier's delay in assisting the fisherman was reasonable. B. The jet skier had no duty to help the fisherman. C. The fisherman was contributorily negligent. D. The delay in assisting the fisherman was caused by the coast guard.

B. The jet skier had no duty to help the fisherman.

A man attended a fund-raising party at the home of a friend. Because the friend was expecting a large crowd to attend, he purchased a number of chairs to use at the party. When the man sat in one of the chairs it collapsed, injuring him. The chair in question was manufactured by a chair company. Analysis of the broken chair revealed that one of the legs of the chair, which the company had purchased from a supplier, had been defectively manufactured such that it could not hold the weight of a normal adult. The company randomly tested the chair legs it purchased from the supplier for strength. Such a test would have revealed the defect in this particular leg. If the man brings a negligence action against the company, which of the following most accurately states what he must show to make a prima facie case against the defendant? A. The man must prove that the chair leg was dangerously defective and that it had not been tested by the company. B. The man must prove that the chair leg was dangerously defective and that the defect would have been discovered but for the company's failure to exercise due care. C. The man must prove that the chair leg was dangerously defective, that it had been tested by the company, and that the tests failed to reveal the defect. D. The man must prove that the chair leg was dangerously defective.

B. The man must prove that the chair leg was dangerously defective and that the defect would have been discovered but for the company's failure to exercise due care.

A doctor and a nurse were having lunch at a fast food hamburger restaurant when the nurse excused himself to get a refill from the soda machine. The restaurant was a franchise and the owner, who was present, also managed the restaurant. A package of ketchup had fallen on the floor when another customer grabbed a big handful of ketchup packages from the bin. The ketchup package was open because several customers had stepped on it. The ketchup remained on the floor for quite some time. The owner was unaware of the spill, and had assigned an employee to clean tables and keep an eye out for spills during the lunch rush. As the nurse was walking past the soda machine, he slipped and fell on the ketchup. When he fell, his head struck the metal table where the beverage dispensers were located. The nurse suffered slight brain damage that resulted in the permanent loss of his sense of smell. The customer who dropped the ketchup knew that the ketchup package was on the floor, and although he could have done so, he did not warn the nurse. In fact, the customer and his friends watched the ketchup spill and laughed out loud when the nurse fell. The nurse asserted a claim against the franchise owner for the injuries he suffered from the fall. Who should prevail? A. The nurse, because the ketchup on the floor constituted an unsafe condition of the premises. B. The nurse, because the ketchup was on the floor for a substantial period of time before the accident. C. The owner, because he assigned an employee to inspect the premises for spills. D. The owner, because the customer could have prevented the injury by warning the nurse about the ketchup on the floor.

B. The nurse, because the ketchup was on the floor for a substantial period of time before the accident.

A health food store sold specialty energy drinks in a variety of flavors and varieties, including gluten-free. One Sunday, a new employee mistakenly filled the gluten-free drink machine with the regular gluten drink mix. A gluten sensitive attorney came in to purchase his standing favorite, a large mocha latte gluten-free drink with a twist of lime. Due to the employee's mistake, the attorney consumed far more gluten than his system could tolerate. Within an hour of drinking the glutenous drink, he was feeling woozy and lapsed into a coma shortly thereafter. He was admitted to the hospital and successfully treated, but he continued to experience nausea and vomiting, which the doctors were hopeful, but not certain, would improve in time. A world-renowned baseball player who was up for contract renewal had been scheduled for a meeting with the attorney, whose specialty was big money professional sports contract negotiations. However, given the attorney's medical problems, the attorney was unavailable to consult with the baseball player as scheduled, and the baseball player found another attorney to negotiate the contract. However, the negotiations broke down and the baseball player was left without a contract. A sportswear company had hired the baseball player to film a series of commercials, pending his contract signing. Based on the player's famous name, the sportswear company had lined up numerous other celebrities to appear in the commercials. When the baseball player announced that negotiations were stalled and that he did not yet have a contract, the celebrities backed out of the deal, causing the sportswear company major financial losses. The sportswear company filed a claim against the convenience store seeking recovery of its financial losses. The jurisdiction applies a comparative negligence standard. Which of the following is the strongest argument in defense of the convenience store? A. The store is not liable for the company's damages, because the company was not a customer of the store. B. The store is not liable, because the store employee's negligence was not the proximate cause of the company's damage. C. The store is not liable for the company's damages, because the gross negligence of the baseball player's attorney is a superseding cause of the company's damages. D. The store is not liable for the company's damages, because the company could have found a replacement baseball player.

B. The store is not liable, because the store employee's negligence was not the proximate cause of the company's damage.

A woman was on her way home from the movies when she heard a loud bang come from a building across the street. When she got to the building, she looked into a smoke filled window and saw someone lying on the floor. The person appeared to be unconscious. She quickly entered and barely pulled a man out of the building before another explosion rocked the structure. The woman suffered almost complete deafness in her left ear and lost sixty percent of the hearing in her right ear. As it turned out, the man was the disowned son of the wealthy building owners and had opened the gas lines before he set the building on fire to get back at his parents. If the woman sues the man for her injuries, what is the likely outcome? A. The woman will prevail, because she prevented injury to the man. B. The woman will prevail, because the man caused the fire that injured the woman. C. The man will prevail, because it was not foreseeable that someone else would enter the building. D. The man will prevail, because the woman assumed the risk of harm by entering a burning building.

B. The woman will prevail, because the man caused the fire that injured the woman.

A woman and a man, who worked for the same company, regularly commuted to work together. Each drove on alternate days. One day when the woman was to drive, she arrived at the man's house at the appointed time. Shortly after they left, the woman informed him that she had received a notice in the mail the day before from the manufacturer of her car warning her that, due to the manufacturer's negligence, there was a defect in her car's brake system and she should not drive the car until the brakes were repaired. The man stated they should return to his house and take his car, but the woman replied that there was a dealership next to their place of work and she would have the brakes repaired during the day. She further stated that she was sure the brakes would hold up on the trip to work. Shortly thereafter, the traffic in front of the woman came to a stop. When she applied the brakes they failed, causing the woman's car to strike the stopped car in front of her. The man was injured in the accident. If the man brings a negligence action against the manufacturer, which of the following most accurately describes the significance of the woman's negligence to the litigation? A. The woman's negligence was the proximate cause of the man's injuries. B. The woman's negligence was a foreseeable danger the risk of which was increased by the manufacturer's negligence. C. The woman's negligence was an independent, intervening force which superseded the manufacturer's liability. D. The woman's negligence was the legal cause of the man's injuries.

B. The woman's negligence was a foreseeable danger the risk of which was increased by the manufacturer's negligence.

A young man decided to celebrate after he received his final grades in law school. Feeling giddy, he consumed two bottles of wine, and then decided it would be a good idea to deep-fry a turkey and have some friends over to his apartment to celebrate. The man heated a pot of oil to 400 degrees and dropped in a ten pound frozen turkey. Within five minutes, the pot of oil and turkey exploded, spreading burning oil across the room. The smoke detector went off, as the apartment filled with deadly smoke. Although he was severely burned, the young man managed to call 911. Within four minutes of the call, an emergency medical response team arrived at the apartment. Because the smoke was so thick inside the house, one of the EMT's sustained a broken leg and had to be taken to the hospital. The EMT asserted a claim against the young man to recover for his injuries. Who should prevail? A. The young man, because he did not intend to harm the EMT. B. The young man, because the firefighter's rule treats emergency personnel as licensees. C. The EMT, because the young man negligently placed himself in a position of peril. D. The EMT, because he succeeded in saving the young man's life.

B. The young man, because the firefighter's rule treats emergency personnel as licensees.

A runner reached first base successfully on an error and then, on the next pitch, attempted to steal second. The catcher threw the ball to the second baseman as the runner slid into second. The runner slid in such a way that his foot was elevated and knocked the ball out of the second baseman's mitt. The second baseman was infuriated by the runner's action and pushed the runner to the ground when he stood up. The runner fell to the ground and suffered a broken arm. It was later discovered that the runner had a calcium deficiency. A person without such a deficiency would not have suffered a broken arm as a result of the push by the second baseman. If the second baseman is liable for causing injuries to the runner, should the damages awarded the runner include compensation for the broken arm? A. Yes, because foreseeability is an element of causation, not damages. B. Yes, because although the broken arm may have been unforeseeable, some harm from being pushed to the ground was a foreseeable type of injury. C. No, because it is not foreseeable that a push would lead to such a severe injury. D. No, because a person who did not suffer from a calcium deficiency would not have suffered a broken arm.

B. Yes, because although the broken arm may have been unforeseeable, some harm from being pushed to the ground was a foreseeable type of injury.

A father and a son were playing catch in their back yard when their neighbor negligently felled a large oak tree. The tree crashed into the yard where the father and son were playing. The son was able to move out of the way, but the tree landed on the father and seriously injured him. In addition to suffering emotional distress at seeing his father injured, the son developed severe and chronic migraine headaches. Under the majority rule, can the son successfully bring a direct claim for negligent infliction of emotional distress against the neighbor? A. Yes, because he witnessed a physical harm occurring to a loved one. B. Yes, because he was in the zone of danger and suffered a physical manifestation of his emotional distress. C. No, because he was not truly at risk of being injured by the falling tree. D. No, because he did not suffer shock resulting from the observance of the accident.

B. Yes, because he was in the zone of danger and suffered a physical manifestation of his emotional distress.

A business woman opened a dance studio in a thriving urban center. Wanting to provide plenty of room for her customers, the business woman hired an independent contractor to remodel the small space and build a stretching bar along one of the walls. One morning, shortly after the grand opening of the studio, the yoga instructor noticed that one of the screws holding the stretching bar was loose, as if it might come unbolted from the wall. The instructor immediately posted a sign stating "Broken - DO NOT USE." The yoga instructor then called the contractor and asked that he come immediately to repair the bar. A few hours later, a new customer entered the studio. While waiting to decide what classes he wanted to join, the customer failed to notice the sign and lifted his leg to try out the stretch bar. Under the customer's weight, the bar broke away from the wall and both the customer and the bar went crashing to the ground. The customer broke his ankle in the fall. If the customer sues the instructor, will he recover? A. Yes, because the placing of the sign was not enough of a remedy to fix a known danger. B. Yes, because the customer was a business invitee. C. No, because the contractor's acts were not the actual and proximate cause of the injuries. D. No, because responsibility for construction and installation of the stretch bar cannot be delegated to the contractor.

B. Yes, because the customer was a business invitee.

The defendant owns a warehouse at the edge of a residential neighborhood. Unfortunately, the warehouse became infested by cockroaches, so the defendant decided to exterminate them. Prior to extermination, the defendant put signs all over the nearby neighborhood warning residents of the upcoming extermination. The defendant took every necessary precaution on the day of the extermination. However, some of the gasses leaked from the warehouse. The wind was especially high that day, and the toxic fumes were carried to the nearby neighborhood. Several of the residents, including two children, became ill from the fumes. The injured residents sued the defendant. Will the injured residents likely succeed? A. Yes, because the defendant breached his duty of care to the residents. B. Yes, because the defendant was engaged in an abnormally dangerous activity. C. No, because the residents will not be able to prove the defendant's negligence. D. No, because the defendant took all possible precautions against injury.

B. Yes, because the defendant was engaged in an abnormally dangerous activity.

During the summer, the plaintiff likes to entertain guests by having dinner parties on his back porch. However, the plaintiff's neighbor, the defendant, has recently started a nightly "cigar club." The club consists of 25 members, and each evening the members meet in the defendant's back yard to smoke a variety of cigars. Due to the number of frequency of the meetings, the plaintiff is unable to continue to host dinner parties because the smoke from the cigar club bothers his dinner guests. Can the plaintiff successfully bring suit against the defendant for private nuisance? A. Yes, because the defendant has caused an unreasonable interference with a right common to the general public. B. Yes, because the defendant's activities substantially and unreasonably interfere with the plaintiff's use and enjoyment of his land. C. No, because the plaintiff has devoted his land to an unusually sensitive use. D. No, because the defendant did not cause any sort of physical intrusion onto the plaintiff's land.

B. Yes, because the defendant's activities substantially and unreasonably interfere with the plaintiff's use and enjoyment of his land.

A man purchased a blender for his wife as a birthday gift. While a friend was visiting the wife, she used the blender to make a smoothie. The blender malfunctioned while the friend was using it, causing a small piece of glass to cut the friend's eye. Is the friend a proper plaintiff in a strict products liability case against the manufacturer of the blender? A. Yes, because the friend was using the blender for its intended purpose. B. Yes, because the friend was using the blender in its defective condition. C. No, because the friend did not purchase the blender. D. No, because the friend was not the owner of the blender.

B. Yes, because the friend was using the blender in its defective condition.

An actor hired a plastic surgeon to perform a facelift. During the surgery, the surgeon became angry at the scrub nurse and slapped her hand away as she tried to hand him a scalpel. The scalpel flew out of the nurse's hand, cutting the actor on the shoulder. The surgeon was able to suture the resulting injury in such a way that the actor was left with no visible scar or permanent injury. The actor filed a battery claim against the surgeon. Will the actor prevail? A. Yes, because the actor suffered an injury, albeit a temporary injury, when the surgeon slapped the scalpel out of the nurse's hand. B. Yes, because the surgeon's intent to make offensive contact with the scrub nurse was transferred to the actor. C. No, because the actor was unconscious during the surgery and unaware of the offensive contact. D. No, because the actor suffered no permanent injury to his shoulder.

B. Yes, because the surgeon's intent to make offensive contact with the scrub nurse was transferred to the actor.

A plaintiff was riding his motorcycle well within the posted speed limit when he looked over at a billboard, which was displaying an illegal risqué picture. As he was looking at the billboard, he failed to see an abandoned truck in the middle of the highway about 200 feet in front of him. Moments later, the plaintiff crashed into the rear of the abandoned truck and suffered injury as a result. If the plaintiff asserts a claim against the owner of the abandoned truck, will the plaintiff recover? A. Yes, for all of his damages, because the defendant created a dangerous condition. B. Yes, but only for a portion of damages, because the abandoned truck was in plain view. C. No, because the plaintiff had the last clear chance to avoid the collision. D. No, because the plaintiff was negligent in failing to keep a proper lookout while driving.

B. Yes, but only for a portion of damages, because the abandoned truck was in plain view.

In a nuisance case, it is a defense that the purported nuisance is more economically valuable than alternative uses of the land. A.True B.False

B.False

A woman invited six friends to her parents' house to watch a football game. The woman's parents were not at home. While they were watching the game, snow fell outside, coating the sidewalk and driveway with a thin transparent layer of very slippery ice. At half-time, all seven people went outside to play in the snow. Most of the people were intoxicated. While fooling around throwing snowballs, one of the friends tripped on the front steps leading up to the house and injured his knee. Which of the following statements is most accurate? A. The woman had no duty of care to her friends. B. The woman had a duty to inspect the premises for unknown dangers and disclose their existence to her friends. C. The woman had a duty to warn her friends of any known dangerous condition on the premises. D. The woman had an absolute duty of care to her friends.

C. The woman had a duty to warn her friends of any known dangerous condition on the premises.

Donald builds on a different lot. To access the back of his new building, he would like to drive a crane across Neighbor Ned's parking lot next door. Neighbor Ned, however, demands rent of $2 million per day for permission. Donald thinks this is outrageous, so one Sunday morning when no one is around, he drives the crane across Ned's parking lot. He does no damage. Ned notices the crane the next day and surmises that it must have driven across his land. Ned sues for trespass. In a bench trial (no jury) applying standard trespass doctrine, the court should rule that: A. Donald is not liable for damages because he did not cause any damage. B. Donald is liable for damages measured by the fair market rental value for the time he used the land. C. Donald is liable for damages measured by the fair market rental value for the time he used the land plus punitive damages in an amount sufficient to deter such behavior in the future. D. Donald is liable for damages in the amount of $2 million unless he can establish that Ned would have accepted a lower number.

C. Donald is liable for damages measured by the fair market rental value for the time he used the land plus punitive damages in an amount sufficient to deter such behavior in the future.

Auntie hasn't seen her nephew Nino, who is now 11, for several years. She arrives just as the school bus is delivering him home. While all Nino's friends laugh and point, she swoops him up in a crushing embrace and places a vast, wet, kiss on one cheek while pinching the other. Nino is seriously embarrassed. He wants your advice: would he prevail if he sued Auntie for battery? A. Nino will lose because Auntie caused him no harm or offense. B. Nino will lose only in a dual intent state, because while Auntie intended to kiss him, she did not intend to harm or offend him. C. Auntie should prevail because an aunt's kiss, pinch and hug is not the sort of touching that reasonable people view as harmful or offensive. D. Auntie should prevail because she reasonably believed that her nephew, not having seen her for all this time, would want to be hugged and kissed.

C. Auntie should prevail because an aunt's kiss, pinch and hug is not the sort of touching that reasonable people view as harmful or offensive.

In tort cases, if a judge determines that the defendant is liable, the judge is most likely to: A. Issue an injunction ordering the defendant to cease the improper behavior. B. Punish the defendants with a fine intended to create incentives to not injure others in the future. C. Award damages based on the actual injury suffered by the plaintiff, without regard to potential injuries suffered by others. D. Balance the benefits of the defendant's actions against the harm suffered by the plaintiff.

C. Award damages based on the actual injury suffered by the plaintiff, without regard to potential injuries suffered by others.

In tort, if a court orders the defendant to pay money, it is most likely: A. Punishment paid to the plaintiff on behalf of the public, as a "private attorney general" B. Punishment paid to the state on behalf of the public. C. Damages paid to the plaintiff to restore the plaintiff to the condition plaintiff was in prior to the tort. D. Damages paid to the plaintiff to restore an injustice.

C. Damages paid to the plaintiff to restore the plaintiff to the condition plaintiff was in prior to the tort.

As the holiday season approached, a homeowner decided to hang lights around his home. He set up a ladder, climbed it, and began to hang the lights along the edge of his roof. As the homeowner reached the corner of his roof he attempted to stretch to attach the lights to the corner rather than reposition the ladder. In so doing, he fell about 12 feet off the ladder. The homeowner felt some pain in his lower back but did not believe he was seriously injured. Shortly thereafter, the homeowner decided he needed more lights to hang on his home. While driving to the store to purchase additional lights, the homeowner purchased a hamburger and began to eat it his car while driving when he was hit from the rear by another vehicle, that had negligently failed to stop. As a result of the collision the homeowner suffered paralysis. The homeowner brought suit against the other driver. At trial, medical experts testified that the homeowner would not have been paralyzed in the collision if he had not fallen off the ladder earlier that day and that the initial back injury could have been treated by any competent practitioner by immobilization in a brace if the homeowner had gone to the doctor immediately. What is the driver's best argument for a reduction in the damages owed to the homeowner? A. Damages should be reduced, because of the doctrine of avoidable consequences. B. Damages should be reduced, because the driver was only passively negligent. C. Damages should be reduced, because the homeowner was comparatively negligent. D. Damages should be reduced, because the driver was not the legal cause of the homeowner's injury.

C. Damages should be reduced, because the homeowner was comparatively negligent.

Dan is large and scary looking, wearing a black leather jacket with gang symbols. He's walking -- or swaggering -- down the street in the dark while tiny Petitcia is coming in the opposite direction. She sees him and is terrified that he intends to mug her or worse. She runs in the opposite direction, falls down and breaks her leg, and then sues him for assault. Dan admits that he dresses this way in order to scare people, pointing out that he lives in a dangerous part of town and that he believes that looking scary is the best way for him to remain safe. He denies, however, that he intended to touch Petiticia in any way. The case is before the judge on cross motions for summary judgment. What result? A. Dan should prevail because he intended no harm. B. Plaintiff should prevail because she was in fear of an imminent harmful touching. C. Dan should prevail even though he intended to put P in fear, because he did not intend to make her fear that he would touch her in a harmful or offensive way. D. Petiticia should prevail because a reasonable person would have feared that Dan intended to mug her.

C. Dan should prevail even though he intended to put P in fear, because he did not intend to make her fear that he would touch her in a harmful or offensive way.

Dan is large and scary looking, wearing a black leather jacket with gang symbols. He's walking -- or swaggering -- down the street in the dark while tiny Petitcia is coming in the opposite direction. She sees him and is terrified that he intends to mug her or worse. She runs in the opposite direction, falls down and breaks her leg, and then sues him for assault. Dan admits that he dresses this way in order to scare people, pointing out that he lives in a dangerous part of town and that he believes that looking scary is the best way for him to remain safe. He denies, however, that he intended to touch Petiticia in any way. The case is before the judge on cross motions for summary judgment. What result? (Remember that on a motion for summary judgment, the judge must take all contested facts in the light most favorable to the non-moving party.) A. Defendant Dan should prevail on both motions because the uncontested testimony is that he intended no harm to Petiticia. B. Plaintiff Petiticia should prevail on Dan's motion because she was in fear of an imminent harmful touching. C. Defendant Dan should prevail on Petiticia's motion because he did not intend to make her expect that he would touch her in a harmful or offensive way, even if he did intend to invoke a more general fear. D. Plaintiff Petiticia should prevail because a reasonable person would have feared that Dan intended to mug her.

C. Defendant Dan should prevail on Petiticia's motion because he did not intend to make her expect that he would touch her in a harmful or offensive way, even if he did intend to invoke a more general fear.

Din owns and operates a jewelry store. He sees a customer pocket a diamond ring and leave the store. Immediately outside the store exit, Din jumps the thief and knocks him down. Unfortunately, the customer hits his head when he falls and is seriously injured. Also, it turns out that Din was mistaken in thinking the customer was a thief: The ring he saw the customer pocket was the customer's, purchased earlier at a different store, which the customer had taken out to compare to those on sale at Din's. The customer, P, sues in battery. P admits that Din's error was reasonable under the circumstances. Common law as set out in Restatement 2d applies. A. P should prevail because the shopkeeper's privilege does not apply where no shoplifting has taken place. B. P should prevail because Din has no privilege to commit battery under the circumstances. C. Din should prevail if he can establish that he did not intend serious injury and that he used reasonable force to recapture a chattel while in hot pursuit. D. Din should prevail because his belief that P had stolen the ring was reasonable.

C. Din should prevail if he can establish that he did not intend serious injury and that he used reasonable force to recapture a chattel while in hot pursuit.

Dirk is checking his email while driving down the highway at 70 mph. He suddenly notices that he is drifting out of his lane and into the way of a rapidly approaching car. He slams on his brakes. A driver in a car behind him is startled and brakes as well; the second car spins out of control, hitting Victoria's new car. Victoria's car is totaled and she was terrified and suffered black and blue marks. She did not require any medical treatment, but she has to replace her car. Victoria sues Dirk in battery. Her complaint states the above facts. Dirk contends that the case against him should be dismissed. (Remember: on a motion to dismiss, the Court must assume that the claims in the complaint are true, and should make reasonable inferences from those facts that favor the plaintiff.) Which is the weakest part of Victoria's case? A. Dirk mainly harmed Victoria's car, not Victoria. B. Dirk did not touch Victoria -- any injury he did to her was indirect. C. Dirk did not intend to touch Victoria -- although he did intend to slam on the brakes. D. Victoria was not physically injured in any significant way.

C. Dirk did not intend to touch Victoria -- although he did intend to slam on the brakes.

Dr Denny, a GP, is sunbathing on Jones Beach when he notices a woman, Pauline, struggling in the water slightly offshore. He jumps into the water and swims out to rescue her. Pauline, however, belongs to a traditionalist religious sect that believes that God determines our destiny in all things, and that it is deeply sinful to interfere with His will. In her view, accepting rescue from an emergency situation like this is deeply sinful and might condemn her to hell. And, as the Book of Mark says, what does it profit a man to gain the whole world if he lose his soul? Accordingly, from the moment when she sees him, Pauline -- who is panicked and has swallowed quite a bit of seawater -- tries to tell him to leave her alone. Dr Denny either does not understand what she is saying or concludes that she is hysterical and doesn't mean it (the testimony is inconsistent). She passes out. He grabs her by the waist and heroically brings her to shore, where he performs mouth to mouth resuscitation and revives her. Once revived, Pauline is furious at having lost her chance of eternal life and somewhat worried that her community will ostracize her (or worse) for having been touched (mouth-to-mouth!) by a strange man. She sues Dr Denny for battery. A. Dr Denny should prevail because, on balance, he did more good than harm to Pauline. B. Dr Denny should prevail because Pauline's beliefs are unreasonable. C. Dr Denny should prevail because reasonable people would not view touching a drowning stranger to rescue her as "harmful or offensive." D. Pauline should prevail because doctors may not touch a patient without the patient's consent, even if the touching would save the patient's life.

C. Dr Denny should prevail because reasonable people would not view touching a drowning stranger to rescue her as "harmful or offensive."

Victor and Victoria are partying in the empty lot next to Dimitri's house. Dimitri, feeling annoyed by their noise and wanting to sleep, says to them: "I have a gun. Stay where you are and don't move or I'll shoot. I'm calling the police." He waves his gun in their general direction from his well-lit porch. Terrified that Dimitri might shoot them if they moved, Victor and Victoria stay still for twenty minutes until they realize that Dimitri has left and his house is dark. Then they head straight to their lawyers' office to file a tort action. Based on the facts above, they are most likely to succeed in a tort action for: A. Battery. B. Assault. C. False Imprisonment. D. These facts are insufficient to state a cause of action.

C. False Imprisonment.

Unknown to the defendant, the plaintiff took a shortcut across the southern tip of the defendant's land each morning on his way to school. Recently, the defendant obtained a pair of snow leopards, which he chained up near the area of the plaintiff's shortcut. One of the snow leopards bit the plaintiff as he was crossing over the defendant's land. If the plaintiff sues the defendant for damages arising from the injuries he sustained, how will the court likely rule? A. For the plaintiff, because the defendant will be strictly liable for the injuries caused by the snow leopards. B. For the plaintiff, because the snow leopards will be considered the equivalent of a mechanical device. C. For the defendant, because the plaintiff was an unknown trespasser. D. For the defendant, because the snow leopards had no known dangerous propensities.

C. For the defendant, because the plaintiff was an unknown trespasser.

Two friends were playing basketball in the park. At one point, the basketball bounced off the backboard and rolled into the street that ran along the park. One of the friends ran after the basketball and ran into the street to retrieve it without first checking to see if cars were coming. The friend ran into the path of a car, whose driver failed to see the friend because the driver was dialing her cell phone at the time. The car hit the friend, causing the friend serious injuries. The friend brought an action against the driver for his injuries. If it is determined that both the friend and the driver were negligent, what, if anything, will the friend recover? A. Nothing, because the collision would never have occurred but for the friend's negligence. B. His total damages reduced by the percentage to which his negligence contributed to his own injuries, but if that percentage exceeds 50%, he will recover nothing. C. His total damages reduced by the percentage to which his negligence contributed to his own injuries. D. His total damages, because the driver had the last clear chance to avoid the collision.

C. His total damages reduced by the percentage to which his negligence contributed to his own injuries.

In a crowded city center, a group of neighbors created a "community garden". Strangers would often stop and help themselves to herbs and vegetables even though the neighbors posted a large sign stating, "KEEP OUT - OUR GARDENING CLUB ONLY". Because thousands of people passed the garden every day, the neighbors hardly ever tasted the fruit of their labor. One of the gardeners was sick and tired of people helping themselves to vegetables from the garden so he rigged a wire that caused a tiny static electrical discharge as a deterrent. The shock was harmless to most people, and did not even harm small children or pets. One day, a tomato thief picked a large ripe tomato. As he put the tomato in his bag, he touched the wire and received a tiny shock. The thief, who had a ten-year-old pacemaker implanted in his chest, suffered a heart attack when the pacemaker malfunctioned from the shock. He died on the spot. The tomato thief's estate brought a wrongful death action against the gardener. Which of the following will the plaintiff likely establish at trial? A. Installation of the shock wire was the legal cause of the death, but not the cause-in-fact. B. Installation of the shock wire was the cause-in-fact of the death, but not the legal cause. C. Installation of the shock wire was the legal cause and the cause-in-fact of the death. D. Installation of the shock wire was neither the legal cause nor the cause-in-fact of the death.

C. Installation of the shock wire was the legal cause and the cause-in-fact of the death.

A man was walking along a city street when he heard cries for help coming from inside a fenced yard. The man noticed a sign on the outside of the fence that said: "Beware! Vicious Attack Dog." The man peered over the fence and saw a boy cowering in a corner of the yard as a large snarling dog approached him. The boy had climbed into the yard on a dare from one of his friends, even though he knew the dog in the yard had a reputation for attacking strangers who entered. However, as the boy climbed over the fence he slipped and severely sprained his ankle so that he could not climb back out of the yard. The man quickly realized that the boy needed help or he would be mauled. The man jumped into the yard and distracted the dog as the boy crawled to a gate. The dog quickly turned and ran at the man, as the boy opened the gate, crawled out, and closed it behind him. The dog jumped at the man just as he leaped out of the yard, severely scratching the man's left leg. The owner had placed signs all around her property warning of her dog because she knew the dog posed a danger to anyone who entered. The man sued the owner for his injuries. What will be the likely result? A. Judgment for the owner, because she was not negligent under the circumstances. B. Judgment for the owner, because the man knew that the dog was dangerous and voluntarily exposed himself to the danger. C. Judgment for the man, because the owner is strictly liable for injuries caused by her animal. D. Judgment for the man, because he was not negligent in rescuing the boy.

C. Judgment for the man, because the owner is strictly liable for injuries caused by her animal.

The defendant has kept a pet lion in his home since the lion was a cub. Since the defendant took the lion in as a pet, the lion has been extremely gentle and has never harmed anyone. When the plaintiff, a friend of the defendant, visited him last weekend, the lion clawed the plaintiff and seriously injured him. If the plaintiff sues the defendant for the injuries caused by the lion, how will the court likely rule? A. Not liable, because the lion had no known dangerous propensities. B. Not liable, because the plaintiff was an invitee on the defendant's land. C. Liable, because the lion is a wild animal. D. Liable, because the lion would not have bitten the plaintiff if it had not been for the defendant's negligence.

C. Liable, because the lion is a wild animal.

A teenager was traveling northbound through an intersection with a green light. At the moment she entered the intersection, two drivers, one traveling eastbound and one traveling westbound, also entered the intersection. Both of these drivers ran red lights as they entered the intersection, and hit the car driven by the teenager at the same moment. As a result of the accident, the teenager suffered a number of serious injuries. In a negligence suit by the teenager against the two drivers, what is the likely result? A. Neither driver will be liable, because the court will apply the but-for test. B. Both drivers will be liable under the but-for test. C. Neither driver will be liable, because the court will apply the substantial factor test. D. Both drivers will be liable under the substantial factor test.

C. Neither driver will be liable, because the court will apply the substantial factor test.

The defendant recently purchased a large, abandoned building in a prime location of the city. The defendant decided to turn the building into upscale lofts, which he can lease for a premium due to the building's prime location. As part of his renovations, the defendant needed to use dynamite to blow up a portion of the building. Prior to dynamiting, the defendant took all of the necessary precautions to prevent against injury. He also obtained all necessary permits from the city. On the day of the dynamiting, the defendant used the explosives at exactly 11:00 A.M. Several blocks away, at the city zoo, a zookeeper was in the lion enclosure for the lion's 11:00 A.M. feeding. When the lion heard the explosion, he became startled and agitated. As a result, he attacked the zookeeper, biting him several times. The zookeeper brought suit against the defendant. Will the defendant be strictly liable to the zookeeper? A. Yes, because the lion is a wild animal and is therefore considered to have dangerous propensities. B. Yes, because dynamiting is considered an abnormally dangerous activity. C. No, because the harm to the zookeeper did not result from the type of danger that justified classifying dynamiting as an abnormally dangerous activity. D. No, because the defendant took all necessary precautions to prevent against injury before engaging in the dynamiting.

C. No, because the harm to the zookeeper did not result from the type of danger that justified classifying dynamiting as an abnormally dangerous activity.

A woman was extremely angry after the judge ruled against her in a child custody hearing. On her way home, she stopped into a bar for a beer. The judge was sitting at a table in the bar with some friends. When the woman saw the judge she screamed curses at him. The judge told the woman that she was an unfit mother and a disgrace to the human race. Enraged, the woman hurled a large beer mug at the judge. The judge ducked in the nick of time but the mug hit the bartender in the head causing him severe brain injury. The bartender filed suit against the judge to recover for his personal injuries. Should the bartender prevail? A. Yes, because the judge's conduct was the proximate cause of the bartender's brain injury. B. Yes, because it was foreseeable that the judge's conduct would result in the bartender's injury. C. No, because the injury was not a foreseeable consequence of the judge's conduct. D. No, because the woman intended to hit the judge, not the bartender.

C. No, because the injury was not a foreseeable consequence of the judge's conduct.

A jogger came across a bicyclist who had fallen from his bicycle and broken his leg. The jogger told the bicyclist that she would go and get help for him. However, the jogger failed to seek help on behalf of the bicyclist. Several minutes later, a man spotted the bicyclist and offered to help him. The bicyclist denied the man's offer because he believed that the jogger had already gone for help. No one else came to help the bicyclist and he was not found for several days, by which time he had suffered severe dehydration. Under the majority rule, is the jogger liable for the injuries suffered by the bicyclist after she failed to assist him? A. Yes, because the jogger had a duty to exercise reasonable care in rendering aid to the bicyclist. B. Yes, because the bicyclist relied on the jogger's promise to his detriment. C. No, because the jogger gratuitously promised to take action to aid the bicyclist. D. No, because a rescuer never has a duty towards the party being rescued.

C. No, because the jogger gratuitously promised to take action to aid the bicyclist.

A woman was jogging on a cold day when she tripped and fell. As a result of the fall, she suffered a concussion and was unable to get up and seek medical attention. A man, who was training for a marathon, jogged past the woman. Unwilling to pause in his training, the man passed the woman without offering her any medical assistance, although he was in a position to easily help her. As a result, the woman suffered hypothermia from laying out in the cold. Does the woman have a cause of action based in negligence against the man? A. Yes, because the man had a duty to act because he could have helped the woman without causing any harm to himself. B. Yes, because the man's failure to act is considered a negligent omission. C. No, because the man had no affirmative duty to take action to aid or protect the woman. D. No, because the man could have placed himself in danger in trying to aid the woman.

C. No, because the man had no affirmative duty to take action to aid or protect the woman.

A man liked to show off his vintage 1969 hot rod. He installed special 22 inch wheels with spinning hubcaps also known as "spinners". The man knew that there were two state laws prohibiting spinning hubcaps and wheels larger than 20 inches. One day, the man stopped at a downtown light with his radio so loud he could not hear anything happening around him. When he stopped, he took the opportunity to light a marijuana cigarette. Both smoking and possession of marijuana are crimes in the state. Meanwhile, a woman was just leaving a local bar. Realizing she was late for an important appointment, she sprinted out of the bar and crashed directly into the man's car. The woman was startled but unhurt. As a result, the woman jumped away from the man's car and was hit by another car. The woman sustained severe injuries to both legs. She filed suit against the man to recover her medical expenses, asserting negligence per se based on the man's violation of at least one state statutes. Is the woman likely to prevail? A. Yes, because as a pedestrian, the woman was a member of the class of persons one of the statutes was designed to protect. B. Yes, because the man had the last clear chance to avoid the harm. C. No, because the man's violation of a statute was not a cause of the woman's harm. D. No, because the woman was injured by another car, not by the man.

C. No, because the man's violation of a statute was not a cause of the woman's harm.

A motorist hits a pedestrian and leaves the pedestrian laying on the side of the road, in need of medical attention. As a result of the accident, the pedestrian suffered broken ribs and a concussion. Shortly after the motorist drove away, a passerby found the pedestrian and robbed him. In the course of the robbery, the passerby broke the pedestrian's arm. Will the motorist be held liable for the pedestrian's broken arm? A. Yes, because the passerby's action was a superseding cause. B. Yes, because the passerby's action was an intervening cause. C. No, because the passerby's action was a superseding cause. D. No, because the passerby's action was an intervening cause.

C. No, because the passerby's action was a superseding cause.

The plaintiff has been seeking medical care from his oncologist, the defendant. Recently, the defendant misdiagnosed the level of the plaintiff's cancer, which impacted the course of treatment prescribed for the plaintiff. If the defendant would have properly diagnosed the plaintiff, the plaintiff would have a 45% chance of survival. However, due to the defendant's malpractice, the plaintiff's cancer is incurable and the prognosis is that that plaintiff will only live for one more month. Under the traditional cause-in-fact rule, will the plaintiff succeed in a malpractice action against the defendant? A. Yes, because the defendant was negligent in properly diagnosing the plaintiff's condition. B. Yes, because the defendant's misdiagnosis decreased the plaintiff's odds of survival. C. No, because the plaintiff cannot prove that, but for the defendant's malpractice, he would have survived. D. No, because even though the plaintiff can successfully establish the prima facie elements of negligence, he cannot prove cause-in-fact by a preponderance of the evidence.

C. No, because the plaintiff cannot prove that, but for the defendant's malpractice, he would have survived.

A technician who had just retired from a demolition company after working there for over twenty years went to visit the site of a demolition being conducted downtown near the river. The technician wanted to see the demolition of a famous office building. Because he had overseen the demolition of many buildings of similar size and construction, the technician knew that debris rarely if ever escaped the blast zone and furthermore, the company would be taking extra care not to permit debris to pollute the river. So the retired technician took a small boat up the river to the demolition site. An employee of the demolition company warned him to stay away from the blast zone but the retired technician told the employee that he knew what he was doing and had on a hard hat. The technician veered very close to the blast zone, closer than a non-employee would have been allowed, and when the building was demolished, a large chunk of cement struck him in the forehead. He was seriously injured as a result. If the retired technician sues the demolition company, will he recover under a theory of strict liability? A. Yes, because the employee should have forbidden the retired technician from getting close to the blast zone. B. Yes, because the demolition company is engaging in an abnormally dangerous activity. C. No, because the retired technician assumed the risk of injury. D. No, because demolitions of buildings are commonplace, theories of negligence rather than strict liability apply.

C. No, because the retired technician assumed the risk of injury.

Pacifico is walking down the street wearing a baseball cap with the brim facing forward. Dimitri, who wears his cap with the brim facing backwards, comes up to P and flicks his finger on the cap brim, causing it to fall off. He then calls Pacifico a yellow-bellied scuzzy solecism, and swaggers off. After picking up his hat, Pacifico contemplates chasing Dmitri and beating him up, as he obviously deserves. However, his friend Leo the Lawyer convinces him to sue instead. He sues for battery. It is uncontested that flicking another man's baseball cap brim is a challenge to a fight, that many men in the neighborhood would immediately respond with violence, and that a man who does not do so is likely to be shamed for his passivity by peers and strangers. What is a court likely to hold? A. Dimitri should prevail because he did not touch Pacifico's person and no harm was done. B. Dimitri should prevail because any offense Pacifico took was inappropriate and the law does not protect the feelings of "snowflakes". C. Pacifico should prevail if he proves that Dimitri flicked the hat intentionally and not by accident. D. Pacifico should prevail only if he can prove, by a preponderance of the evidence, that his hat was damaged in some way.

C. Pacifico should prevail if he proves that Dimitri flicked the hat intentionally and not by accident.

When Professor Greenwood refers to the Cain Principle, this is a reference to and shorthand for: A. Citizen Kane - a classic movie about a fictionalized William Randolph Hearst, the (in)famous yellow journalist and war-mongerer, in which a search for the deep meaning of a mysterious phrase ultimately discloses that even a seemingly cynically ambitious and cruel person has a childhood hurt. It is shorthand for the idea that "to know is to forgive": an author (or a lawyer) can get you to empathize with anyone. B. Cane sugar: inside the bitter is the sweet. Shorthand for the idea that there are two sides to every story; don't jump to conclusions that one is right and the other wrong. C. The biblical story of Cain and Able. Cain murdered his brother and defended himself on the principle, "Am I my brother's keeper?" The phrase is, therefore, shorthand for the claim of many tortfeasors that they should have no responsibility for the consequences of their actions and that, instead, tort victims should look after themselves. Of course, it is also a rhetorical attack on that position, since it analogizes the tortfeasor to a murderer trying to avoid responsibility. D. The biblical story of Cain and Able and Cain's comment "Am I my brother's keeper?" Cain, here, represents the individualist tradition in American politics. We, after all, are mostly descended from immigrants who fled the all-encompassing mutual responsibilities of traditional villages or farming communities abroad or in the US, and we value the freedom and irresponsibility of anonymity, whether on the streets of a city or the internet.

C. The biblical story of Cain and Able. Cain murdered his brother and defended himself on the principle, "Am I my brother's keeper?" The phrase is, therefore, shorthand for the claim of many tortfeasors that they should have no responsibility for the consequences of their actions and that, instead, tort victims should look after themselves. Of course, it is also a rhetorical attack on that position, since it analogizes the tortfeasor to a murderer trying to avoid responsibility.

A baker had skied since she was a child and was very familiar with the mountain closest to her home. One day, while on the mountain, the baker was crossing the slope with wide turns to diminish her angle of descent. At the same time, a former Olympic gold medal skier, who was a ski instructor and member of the ski patrol, was speeding straight down the hill. The gold medal skier saw the baker midway across the hill in one of her traverses, and attempted to ski behind her. However, the gold medal skier underestimated the baker's speed and ran directly into her, fracturing the baker's knee. The gold medal skier's conduct will be judged against which of the following standards? A. The conduct of a reasonably prudent person. B. The conduct of a reasonably prudent person in an emergency. C. The conduct of a reasonably prudent person with superior skiing knowledge and expertise. D. The conduct of a reasonably prudent person with the same age, knowledge, and experience.

C. The conduct of a reasonably prudent person with superior skiing knowledge and expertise.

Defendant Driver had a sudden aneurysm while driving. Deprived of blood flow to his brain, he lost consciousness. With a heavy foot on the gas pedal, he plowed the car off the street and into a school playground, severely injuring 4 children. By the time the car came to a halt, Driver was dead; it is not clear whether he was dead when his car hit the children. The victims sue his estate (and, indirectly, his insurance company). A. The court should instruct the jury that defendant is liable if he failed to exercise the care that a reasonable driver would under the circumstances. B. The court should instruct the jury that Driver was only obligated to exercise the care that a reasonable person would exercise after suffering a fatal aneurysm. C. The court should dismiss the case because the defendant breached no duty. D. The court should grant judgment for plaintiffs because no rational jury could find that Driver was exercising the care that a reasonable person would under the circumstances, since he was either unconscious or dead.

C. The court should dismiss the case because the defendant breached no duty.

An avid gardener specialized in carnivorous plants. The crowning glory of her garden was a towering trellis, covered with pitcher plans and Venus fly traps, which abutted her neighbor's backyard. The neighbor had a 10-month-old son who loved to play in the backyard and had already started to walk. The son was extremely allergic to ragweed. Unknown to the gardener, a patch of ragweed had taken root in her garden. Given all the other varieties of plants, the gardener failed to notice the ragweed. One afternoon, while playing in the backyard, the son brushed up against the gardener's trellis and came in contact with the ragweed. Within two hours, his skin had broken out in hives, and he started to wheeze. His mother treated him with an antihistamine cream and told him not to scratch. She intended to take him to the doctor in the morning, if necessary, where the doctor could give him a cortisone injection to stop the allergic reaction. During the night, however, the son's breathing became labored and he went into anaphylactic shock, a severe allergic reaction that can cause death if not treated. The next morning when his mother came in to check on her son, the mother was horrified to find her young son unconscious in his bed; he has stopped breathing several times during the night. She took him to the emergency room, only to learn that the son was an asthmatic, a medical condition which the son's pediatrician had not yet had reason to discover. The son was admitted into the children's intensive care unit, where he remained for several weeks. Although he eventually recovered, the mother incurred thousands of dollars in unreimbursed medical expenses and lost wages. The mother filed suit against the gardener to recover her financial damages. Which of the following is the gardener's best defense? A. The gardener should not be held liable, because the doctor's negligence in failing to discover the son's asthma constitutes a superseding cause. B. The gardener should not be held liable, based on the eggshell plaintiff rule, because the son was unusually susceptible to ragweed and an asthmatic. C. The gardener should not be held liable, because she breached no duty to the mother or her son. D. The gardener should not be held liable, because she had no reason to know that the son was allergic to ragweed or was asthmatic.

C. The gardener should not be held liable, because she breached no duty to the mother or her son.

A famous golfer lived in a gated community in an upscale suburban area. In order to enter the community, any non-resident had to be on a list of people identified as being allowed to enter. The security guards were hired by a security company, which was contracted and paid for by the residents of the community. The golfer found it essential to live in such a community with a guarded entrance because he was involved in a public sex scandal and wanted to protect his privacy. In particular, there were many people who wanted to see him injured. The golfer conveyed this information to the guards, and made it absolutely clear that no one was to enter the property on his behalf without his express written permission. One night, the golfer was at home alone working on his putting game when he heard a knock at his front door. When the golfer opened his door he found himself facing the barrel of large gun, which was held by one of his ex-girlfriends. Apparently, the guards at the front gate had become lazy about checking the list of persons allowed into the community, and had allowed the irate ex-girlfriend to enter without confirming she was on the list. Several minutes after the ex-girlfriend began beating the golfer with a golf club, the security guards arrived and removed the ex-girlfriend from the premises. As a result of the beating, the golfer's hand was severely injured. The golfer brought an action against the security guard company for his injuries. Who should prevail? A. The company, because the golfer is limited to contractual damages based upon the agreement with the security guard company. B. The company, because the ex-girlfriend's actions were a superseding cause of the golfer's injury. C. The golfer, because the security guard's failure to check whether the ex-girlfriend was on the list increased the risk that the golfer would be harmed. D. The golfer, because the security guard company is a common carrier who owes a higher degree of care to the golfer.

C. The golfer, because the security guard's failure to check whether the ex-girlfriend was on the list increased the risk that the golfer would be harmed.

A scooter manufacturer had been making scooters for two generations. The family of the company's founder took great pride in the quality of the scooters and personally supervised the assembly line several hours a day. Despite careful inspections, the company was unaware that the scooters' handlebars had some issues. The components of the handlebars purchased from a supplier were problematic. The handlebars could get stuck without warning, making the scooter impossible to steer. A man bought one of the scooters from a local retailer. The man took the scooter out for a ride. As he made his first turn onto a busy street, the handlebars get stuck. The man rode straight into traffic and was severely injured. The man sued the local retailer and the manufacturer. Which of the following is the most likely outcome of the suit? A. The man cannot recover against the retailer, because the retailer only sold the scooter. B. The man will not prevail against the manufacturer, because the manufacturer exercised due care and the faulty handlebars were the fault of the supplier. C. The man will prevail against both the retailer and the manufacturer, regardless of whether the manufacturer broke the handlebars or the handlebars came that way from the supplier. D. The man will not prevail against the retailer, but he should prevail against the manufacturer, because they made the scooter.

C. The man will prevail against both the retailer and the manufacturer, regardless of whether the manufacturer broke the handlebars or the handlebars came that way from the supplier.

A woman was attempting to connect the cable television from her house to the antenna when she suffered a severe electrical shock because she failed to turn off the power switch. The woman fell to the grass, she stopped breathing and her heart went into ventricular fibrillation. A retired nurse lived next door, saw the woman's accident and realized that she could be revived by immediate application of an automatic external defibrillator or AED. The nurse had an AED and was trained in how to use it. However, the nurse never liked the woman because her dog barked constantly so she did nothing. By the time an ambulance arrived eight minutes later, the woman was dead and could not be revived. The woman's son and sole survivor brought an appropriate wrongful death action against the nurse. Who should prevail? A. The son, because the nurse knowingly permitted the woman to die when she could have saved her and failed to act out of spite. B. The son, because a reasonable person in the same circumstances would have come to the woman's aid. C. The nurse, because she owed no duty to the woman. D. The nurse, because there was no reasonable certainty that the AED would have revived the woman.

C. The nurse, because she owed no duty to the woman.

A dog owner was surprised to hear from her mailman that her small dog had bitten him when he was delivering the mail. The owner assured the mailman that her dog must have been very upset to do so, because she was ordinarily such a well-behaved little dog. Four days later, a neighbor and her daughter were selling cookies door-to-door. The daughter was not used to dogs. She came up to the dog and tried to pick it up. The dog immediately snapped at her, biting her face and ear. The owner told her neighbor that it was the daughter's fault for trying to pick up the dog. When the daughter turned out to need extensive cosmetic surgery to repair the injury, the neighbor filed a strict liability action against the owner for the daughter's injuries. What must the neighbor prove to prevail in her strict liability claim? A. The owner was negligent in not leashing the dog. B. The daughter did not try to pick up the dog. C. The owner knew that the dog had a tendency to bite. D. The neighbor and daughter were invited guests.

C. The owner knew that the dog had a tendency to bite.

A man was upset because he heard that his friend had been spreading rumors behind his back. The man worked himself into a frenzy and approached his friend. Before the friend could say anything, the man pushed him to the ground. The friend fell on a rusty nail and later died of a blood infection resulting from being cut. The friend's wife sued the man for damages resulting from the friend's death. Is the man liable for the death of his friend? A. No, because he did not intend to cause his friend's death. B. No, because he was unaware of the rusty nail. C. Yes, even if he had no desire to kill the friend. D. Yes, because the death of his friend was a foreseeable consequence of his actions.

C. Yes, even if he had no desire to kill the friend.

The defendant is a manufacturer who designed and manufactured a utility knife that is owned by the plaintiff. The utility knife contains both a sharp knife and a small pair of scissors. The utility knife was packaged in a box which contained a warning about the danger of using the scissors, but made no mention of using the small knife. Without realizing how sharp the knife was, the plaintiff opened it and cut his finger. The plaintiff now claims that the utility knife was dangerously defective because it contained an inadequate warning. Is the plaintiff correct? A. The plaintiff is correct, because the warning label failed to mention all dangers associated with the utility knife. B. The plaintiff is correct, because the warning label should have been contained on the product itself, not on the box. C. The plaintiff is incorrect, because the utility knife is so obviously dangerous that a warning as to its danger is unnecessary. D. The plaintiff is incorrect, because an inadequate warning label cannot render a product dangerously defective.

C. The plaintiff is incorrect, because the utility knife is so obviously dangerous that a warning as to its danger is unnecessary.

A film production company was on location filming a movie about a grizzly bear. During scenes in which the bear did not appear, the bear was kept in a locked cage. However, most of the cast and crew knew that the bear was very friendly to people and easy to work with. One afternoon, some crew members were eating lunch near the bear's cage. The crew members were unaware that the bear had not yet been fed and was slightly cranky. When one of the crew members approached the cage to pet the bear, as he often did, the bear swiped at him through the bars of the cage, injuring him severely. The crew member filed suit against the production company on a theory of strict liability. Assume this jurisdiction has adopted a contributory negligence statute. Which of the following is correct? A. The production company is strictly liable, because the company did not take proper precautions to prevent the bear from swiping at people through the bars of the cage. B. The production company is strictly liable for the crew member's injuries, because the bear was a wild animal. C. The production company is not strictly liable for the crew member's injuries, because the crew member was contributorily negligent. D. The production company is not strictly liable for the crew member's injuries, because the bear was not generally dangerous.

C. The production company is not strictly liable for the crew member's injuries, because the crew member was contributorily negligent.

A man was fired from his job as a banker. He was fixated on retaliating against his former boss. One day as the bank was about to close, the man entered his boss's office with a homemade bomb. He threw the bomb into the office intending to injure his former boss but not intending to injure the secretary, who was always present in the office. The bomb went off and injured both the former boss and his secretary. The secretary filed a lawsuit against the man for tortious battery. Who will prevail? A. The man, because he did not intend to injure the secretary. B. The man, because he did not intend to cause the consequences of his act. C. The secretary, because there was a substantial certainly that the bomb would cause her injury. D. The secretary, because the man intended to cause the consequences of his act.

C. The secretary, because there was a substantial certainly that the bomb would cause her injury.

The defendant suffers from a mental disability that diminishes his capacity to make rational decisions. The defendant attempted to assist the plaintiff in an emergency situation, and as a result of the defendant's actions, the plaintiff suffered additional injuries. To what standard of care will the defendant be held? A. The standard of care of a reasonably prudent person. B. The standard of care of a reasonably prudent person with diminished mental capacity. C. The standard of care of a reasonably prudent person acting in the emergency situation. D. The standard of care of a reasonably prudent person with diminished mental capacity acting in the emergency situation.

C. The standard of care of a reasonably prudent person acting in the emergency situation.

A businessman and a doctor are next-door neighbors who had been feuding for many years. One morning, the businessman started his car, intending to back out of his driveway, when he noticed that his trash cans were placed in his driveway blocking his exit. Suspecting that the doctor had placed the trash cans in his path, the businessman ran into the doctor's yard and started pulling out decorative plants that the doctor had recently planted. The doctor emerged from his house, saw what the businessman was doing, grabbed a baseball bat, and chased after the businessman. The businessman ran back into his yard with the doctor waving the baseball bat closely behind. The businessman's wife came out of the house when she heard the commotion. She screamed at the doctor to stop, but the doctor looked at her and snarled, "Get back in the house! This does not involve you!" The doctor continued to chase the businessman to the wife's horror. The businessman then tripped, and as he lay on the ground, the doctor hit him with the baseball bat, breaking the businessman's arm. Terrified by the course of events, the wife fainted. The wife now suffers recurring nightmares reliving the incident. If the wife sues the doctor for intentional infliction of emotional distress, what will be the likely outcome? A. The doctor will win, because the wife was not within the zone of danger. B. The doctor will win, because he did not intend to inflict any injury to the wife. C. The wife will win, because the doctor hit the businessman and knew the businessman's wife would see the event. D. The wife will win, because she contemporaneously observed the entire incident.

C. The wife will win, because the doctor hit the businessman and knew the businessman's wife would see the event.

The owner of a gift store saw a woman leaving his store. He noticed that the woman appeared to be concealing an expensive gift item under her coat. The owner told the woman that she had to accompany him to the store office. Not knowing why the owner wanted her to accompany him, the woman followed him without saying a word. The owner escorted the woman to the office, asked her to enter, and then, while he remained outside the office, he locked the door once the woman was inside. The woman heard the owner lock the door, but decided to sit quietly for a few minutes until the owner returned. The owner then went to check his inventory to see if anything was missing. A cashier who had viewed the incident then approached the owner and explained that the woman had paid for the item in question. The owner instructed his cashier to release the woman from the office and offer his apologies for the mix-up. The owner then left for lunch. Before the cashier was able to unlock the door to the office, the store became very busy, requiring the cashier to tend to the cash register. The cashier subsequently forgot that the woman was locked in the office. After approximately one hour the woman climbed out the office window and went home. If the woman sues the owner for her confinement in the office, what will be the likely result? A. The owner will win, because he is privileged to detain a person if he has a reasonable belief the person has committed a theft. B. The owner will win, because the woman suffered no injury as a result of the confinement. C. The woman will win, because the owner went to lunch without telling the woman that she was free to leave. D. The woman will win, because she did not actually steal the merchandise in question.

C. The woman will win, because the owner went to lunch without telling the woman that she was free to leave.

Daraugh sees a car parked near her home with the keys still in it. She "borrows" the car to do some errands, drives it around for a day, fills the gas tank and returns it to the parking spot. No damage is done and, indeed, the owner, Owen, does not even realize that his car was used by a stranger until a month later when he receives an inquiry from a friend in the vicinity who noticed Daraugh taking it and thought it odd. Owen wants to sue Daraugh. Owen is most likely to prevail if he brings a cause of action in: A. Trespass. B. Conversion. C. Trespass to chattels. D. Owen has no damage and therefore no cause of action. E. Owen has no cause of action because he acted unreasonably in leaving the keys in the car.

C. Trespass to chattels.

Same facts, except that the car's owner is Zipcar, which is in the business of renting its cars out for short periods of time. There is no evidence that the Zipcar lost any business - as far as its evidence shows, the car would have sat parked during the time that Daraugh used it. Zipcar is most likely to prevail if it brings a cause of action in: A. Trespass. B Conversion. C. Trespass to chattels. D. Zipcar has no damage and therefore no cause of action.

C. Trespass to chattels.

In a nuisance case, it is a defense that the defendant is using its land in a lawful manner and maintains its activities on its own land. A. True B. True, but only if its activities do not physically invade the surrounding land, such as by smell or sound. C. True, but only if activities do not impact the surrounding land in a way that materially diminishes plaintiff's enjoyment of its property or materially diminishes the market value of plaintiff's property. D. False

C. True, but only if activities do not impact the surrounding land in a way that materially diminishes plaintiff's enjoyment of its property or materially diminishes the market value of plaintiff's property.

An office assistant leased a used car from a car dealer. While driving the car, which was a gas/electric hybrid, the office assistant lost control of the car in a driving rain storm and crashed head-on into a tree. An emergency crew was dispatched in response to a call from a witness who said that the driver of the car was injured, but the witness was unable to open the car's door. The emergency crew arrived and used specialized hydraulic equipment to cut through the car's door. Unfortunately, although it did not make a sound, the hybrid car was still running and the operator of the hydraulic equipment received a nasty electric shock that sent him to the hospital. It was later determined that the car model had a design defect. Under modern trends, does the emergency worker have a product liability action against the car dealership that leased the car? A. No, because a commercial lessor is not liable for the lease of defective used goods. B. No, because the emergency worker was not the consumer of the product and is therefore not a proper defendant. C. Yes, because the emergency worker is a reasonably foreseeable plaintiff. D. Yes, because the car dealership was negligent in leasing a defective car.

C. Yes, because the emergency worker is a reasonably foreseeable plaintiff.

An engineer operated a satellite network control center. Various companies throughout the world hired the control center to maintain their spacecraft and to provide customer service and troubleshooting services when their systems go down. Due to the rapid increase in satellite use for telephone, radio and television data, it was common for a client's data stream to crash at least once a month. When such a crash occurred, the client's network was usually up and running within three to five minutes. One day, the control center's most experienced employee announced that he was quitting. The engineer hired an independent contractor until he found someone to permanently replace the experienced employee. The independent contractor was assigned to the former employee's desk, and like everyone else, the contractor received his daily assignments from the engineer. The contractor's work was subject to inspections by the quality control manager. After two-weeks, the contractor was assigned to maintain the spacecraft of one of the control center's most valued customers. In February, due to the independent contractor's negligence, the client's spacecraft lost the television signal during the Superbowl. It took two weeks to repair the damage and to return the client's satellite to its nominal operating condition. Can the engineer be held liable for the damages incurred by the client? A. No, because the client assumed the risk of a crash. B. No, because the independent contractor was not an employee. C. Yes, because the independent contractor was treated like other employees. D. Yes, because the control center provided services to maintain satellite networks.

C. Yes, because the independent contractor was treated like other employees.

A realtor bought a house in a developing neighborhood at a foreclosure sale. She planned to fix up and sell the house for considerable profit. The realtor hired a roofer to install a new roof. The roofer was a friend of the realtor's brother. The roofer agreed to re-roof the house for $8,000 if the realtor would allow him to put a sign in the front yard of the house advertising the roofer's business. The usual cost for a new roof was $8,500. The realtor chose a color and told the roofer, "Do the best job you can." The realtor paid the roofer $4,000 in advance for materials. As the roofer was installing new tiles near the apex of the roof, he accidentally dropped a hammer. The hammer hit a painter in the head, causing the painter to lose sight in one eye. The painter, who was working on the property at the request of the realtor, asserted claims against the realtor and the roofer for his injuries. The painter recovered $100,000 in damages from the realtor under a theory of vicarious liability. Does the realtor have any recourse against the roofer? A. No, because payment by the realtor constituted partial acceptance of the work. B. No, because the realtor contracted with the roofer to do the work. C. Yes, because the judgment against the realtor was based on vicarious liability. D. Yes, because the realtor's conduct was not the actual or proximate cause of the harm to the painter.

C. Yes, because the judgment against the realtor was based on vicarious liability.

A computer programmer became despondent after he was given two weeks notice at his job. The programmer had given ten years of loyal service to his employer. The programmer decided to kill himself by ingesting poison. He ordered cyanide from a chemical supplier, mixed the deadly chemical with fruit punch in a glass, and prepared to drink it at his desk in order to maximize the impact on his employer. A co-worker, who happened to be near the programmer's office, noticed a bottle with a skull and crossbones on the label and figured out what the programmer was doing. The co-worker rushed into the office, and yelled, "Stop!" The programmer changed his mind about committing suicide and decided that he wanted to live. At that moment, however, the co-worker lunged at the programmer and grabbed the glass of poisoned punch. The co-worker wrestled with the programmer, the glass broke cutting the co-worker, and the poison entered his blood stream. As a result, the co-worker suffered mild but permanent brain damage. A statute in the jurisdiction makes attempted suicide a crime. The co-worker asserted a claim against the programmer to recover damages for his injuries. Should the co-worker prevail? A. No, because the programmer did not intend to harm himself. B. No, because attempted suicide was a criminal offense in the jurisdiction. C. Yes, because the programmer put himself in a position of peril. D. Yes, because his actions saved the programmer's life.

C. Yes, because the programmer put himself in a position of peril.

Many families loved to watch their neighborhood pick-up football games. According to the neighborhood tradition, whenever a player scored a touchdown, the player would immediately rush full-speed at one of the fans on the sidelines and jokingly tackle him or her. One Saturday, many families were enjoying a game when a player scored a touchdown. He immediately looked for his little cousin on the sidelines, intending to tackle him. His uncle, standing directly behind the player's cousin, saw the player looking in their direction. Exhilarated by the play, he jokingly called out, "You may be big but you still aren't big enough to take down your uncle yet!" The player charged at him. The uncle tried to move out of the way, but the player succeeded in tackling him. The uncle fell and sustained a concussion. The uncle sued the player for battery, and the player asserted the defense of consent. Will the player's defense be successful? A. No, because the uncle did not intentionally consent to the player's contact. B. No, because the uncle did not expressly consent to the player's contact. C. Yes, because the uncle consented to the contact. D. Yes, because the player reasonably believed that the uncle had consented to the contact.

C. Yes, because the uncle consented to the contact.

A prominent animal rights activist and his friend, a strict vegetarian, went out to dinner at a Mexican restaurant the vegetarian favored. As usual, he placed an order for black beans and rice, his favorite meatless entrée. At the end of the meal, when the waiter brought him his check, the vegetarian remarked, "Thank you, that was absolutely delicious." The young waiter grinned proudly. "Yes, that pork seasoning makes all the difference," he said. "I added it especially for you." At that point, the vegetarian recognized the waiter as his son's classmate, whom his son had just beaten out for the last slot on the high school's extremely competitive varsity baseball team. Convinced that the waiter had purposely added pork to the entrée in order to sabotage his strict vegetarian code, the vegetarian filed a lawsuit against the waiter for battery. Is the vegetarian likely to prevail? A. No, because the waiter did not come into contact with the vegetarian's body. B. No, because by entering the restaurant and ordering food, the vegetarian consented to the waiter's contact with his food. C. Yes, if the waiter knew that the man was a vegetarian. D. Yes, if the waiter deviated from the standard preparation of the menu item.

C. Yes, if the waiter knew that the man was a vegetarian.

An accident occurred at a construction site. The shop foreman directed a young laborer to shovel mortar into a bucket. As the bucket was lifted to the third floor, the pulley collapsed and the bucket fell on the laborer's leg. After a trial on the merits, a jury returned a verdict finding that the laborer's injuries were caused by negligence on the part of the construction company, the pulley manufacturer, and the laborer. The jury assigned 51 percent of the fault for the laborer injuries to the laborer, 19 percent to the construction company, and 30 percent to the pulley manufacturer. The jury found that the amount of the laborer's injuries was $300,000. The attorney for the laborer enforced the judgment against the construction company and collected $147,000 from the construction company. If the construction company brings an action against the pulley manufacturer for contribution, how much is the construction company entitled to collect? A. $0. B. $33,000. C. $57,000. D. $90,000.

D. $90,000.

The defendant is the manufacturer of Roman Candle fireworks. The defendant packages the Roman Candles in a package which merely states, in very small font, "may be hazardous and cause burns" in the corner of the package. Assume that the defendant markets its products to children age 14 and under. The plaintiff's son, a 12-year-old boy, is badly burned when using one of the Roman Candles. The plaintiff sues the manufacturer on behalf of her son on a products liability theory, alleging absence of adequate warnings. Will the plaintiff likely prevail? A. Yes, because the warnings were inadequate to warn a user of the dangers of the product. B. Yes, because the warnings were inadequate to warn a child of the dangers of the product. C. No, because the product was accompanied by a warning. D. No, because the fireworks are so obviously dangerous that a warning is unnecessary.

D. No, because the fireworks are so obviously dangerous that a warning is unnecessary.

In the midst of an unexpectedly severe storm, defendant D's car broke down outside of P's rural home. A nearby stream was rapidly rising and D realized that when it rose above its banks, which was likely to happen shortly, the flood waters would leave her trapped in the car, and possibly worse, since the car might well be swept away. D realized she needed to find shelter immediately to avoid mortal danger. Accordingly, she knocked on the door to P's house, and when she realized that it was empty and likely uninhabited, she broke a window and climbed in. P sues to recover for the damage D did to the house. Which of the following is the best statement of the applicable rule of law? A. Human life is more important than property, so D was entitled to break into the house out of private necessity. B. Necessity can justify using another's property, but it never justifies damaging property not your own. C. In an emergency, it may be reasonable to take less care than in calmer circumstances. D. In an emergency, one may use another's property, but the actor must pay for any damage that results.

D. In an emergency, one may use another's property, but the actor must pay for any damage that results.

Alice has Alzheimer's. She is disinhibited, a frequent symptom of the disease. One day, Alice's volunteer caretaker was changing her diaper. Not recognizing the caretaker and offended at being treated like a baby, Alice thrashed about, injuring the caretaker. The caretaker sues Alice in negligence. The court rules that an Alzheimer's sufferer is held to the same standard of reasonable care as everyone else, and it is not a defense that Alice was disinhibited or incapable of correctly understanding the situation.Which of the following rationales best explains that holding in a way consistent with tort norms? A. Even though the patient is innocent, in the sense of having no culpable mental state, she caused the accident and therefore is the more appropriate cost-bearer. B. Holding the patient liable is a way of signaling to responsible persons - the patient's guardian or heirs - that the guardian has a duty to ensure that the patient does not cause injury. C. It eliminates incentives for defendants to fake mental illness which courts and juries may have trouble identifying. D. It forces persons with disabilities to pay for the damage they do if they are to live in the world.

D. It forces persons with disabilities to pay for the damage they do if they are to live in the world.

A professional tennis player bought herself a convertible after winning the U.S. Open. The tennis player and her sister always talked about riding in a convertible when they were little girls, so she often took her sister for rides in her new convertible. One day, the sister asked to drive the convertible. The tennis player knew her sister was an experienced driver, so she agreed and got into the passenger seat. As the women were riding down a steep hill, the convertible's brakes failed, and they crashed into tractor-trailer moving through the intersection at the bottom of the hill. The crash killed the sister and seriously injured the tennis player. An investigation revealed that a faulty brake caliper had caused the brakes to fail. The convertible manufacturer had gotten the calipers from a supplier and put them through an extensive testing program. The records from the tests showed no defects in this pair of calipers. The sister's estate sued the supplier for the defective caliper using a products liability negligence theory, and proved that the convertible manufacturer didn't properly test the caliper. What effect would the failure to properly test have? A. It would relieve the supplier from liability, because the manufacturer did not avail itself of the last chance to avoid the harm. B. It would relieve the supplier from liability, because the supplier reasonably relied on the fact that the manufacturer would test the caliper. C. It would relieve the supplier from liability, because the manufacturer is in privity with the customer. D. It is irrelevant to the claim.

D. It is irrelevant to the claim.

One afternoon while skiing on a resort mountain, a young man fell and badly hurt his back. The ski patrol rushed to the young man's aid, loaded him onto a gurney, and transported him to the ski lodge. They called an ambulance to transport the man to the hospital. When the ambulance arrived, the ambulance driver directed the man to sign a document agreeing not to sue the owner of the ambulance, for any injuries he might incur as a result of the drive to the hospital. The young man read and signed the document. While driving to the hospital, the ambulance driver lost control of the ambulance because he was driving too fast for the icy road conditions. The ambulance left the road and hit a tree just off the roadway. The young man suffered a broken arm in the collision. If the young man brings suit against the ambulance service for the broken arm suffered in the collision with the tree, what will be the effect of the document he signed? A. It is an express assumption of risk, which will bar the young man's suit against the ambulance service. B. It will bar the young man's recovery if he was aware of its terms. C. It will not bar the young man's recovery, but only if the accident was not within the scope of the risks the young man consented to assume. D. It will not bar the young man's recovery if the ambulance service was the only one available in the area.

D. It will not bar the young man's recovery if the ambulance service was the only one available in the area.

A thief made his living by stealing other people's cattle and selling the meat to unscrupulous butchers. He drove his truck and trailer to a rancher's ranch one evening because he had heard that the rancher was famous for his cattle. The thief set up his trailer and climbed into a field intending to steal a cow. Unknown to the thief, the field contained the rancher's prize fighting bull, used in bullfighting. The bull charged the thief and knocked him to the ground, breaking his ankle in the process. A driver who happened to be driving by the rancher's property saw the thief fall and, knowing of the rancher's bull, realized that the thief was in extreme danger. The driver stopped his truck, got out and vaulted the fence into the field. Waving a red bandanna to distract the bull, the driver shouted to the thief to get out of the field while the driver would draw the bull off. The bull charged the driver, who spent an exciting few minutes dodging the enraged animal while the thief hobbled to the fence and climbed over. As the driver abandoned his bandanna and leaped over the fence, his foot caught on the boards and he fell out of the field, breaking his leg. The driver sued the thief for his injuries. What result? A, Judgment for the thief, because the driver knew that the bull was dangerous and voluntarily exposed himself to the danger. B. Judgment for the thief, because the driver's decision to intervene was an abnormal response to the situation. C. Judgment for the driver, because he prevented the thief from suffering almost certain death or severe injury from the bull. D. Judgment for the driver, because the thief negligently or intentionally placed himself in peril.

D. Judgment for the driver, because the thief negligently or intentionally placed himself in peril.

What is the source of the obligations enforced by tort law? A. The parties' own consent. B. Statutes enacted by the legislature or regulations enacted under its authority. C. Natural law theories regarding the rights one acquires by mixing ones labor with the products of nature. D. Judicial views of social norms that judges believe they can discover.

D. Judicial views of social norms that judges believe they can discover.

In the midst of a large storm, Ned notices that the rising flood waters seem likely to wash his parked car into his neighbor Homer's yard, which would cause serious damage to Homer's beloved and valuable rose garden and quite likely smash into Homer's house, endangering the physical safety of anyone in the house. He realized he could easily and safely save Homer's property and eliminate the physical danger by moving his car. However, he decided, instead, to stay in the warm comfort of his home; after all, he thought, his car had virtually no economic or sentimental value and the rose garden and home were, in the end, more Homer's problem than his. The floodwaters did indeed wash his car into Homer's yard and home, causing quite a bit of avoidable damage. Homer sues Ned to recover the damage that resulted from Ned's decision. A. Ned is entitled to a sudden emergency instruction. B. Ned is liable for negligence because he failed to exercise the care that a reasonable person would under the circumstances. C. Ned is liable for conversion because he intentionally deprived Homer of his property. D. Ned is not liable in negligence because he owes no duty to Homer under the circumstances, but he might be liable in trespass because his car invaded Homer's property.

D. Ned is not liable in negligence because he owes no duty to Homer under the circumstances, but he might be liable in trespass because his car invaded Homer's property.

The defendant is playing with a soccer ball in his yard when he kicks the ball into the yard of the house next door. The house next door is rented, but not owned, by the plaintiff. The defendant is under the mistaken belief that the property onto which he kicks the ball is part of the land owned by the defendant. The defendant himself does not enter onto the plaintiff's land. If the plaintiff sues the defendant for trespass, what defense is likely to succeed? A. The plaintiff did not actually own the land in question. B. The defendant did not actually enter onto the land in question. C. The defendant did not know that the land in question was not owned by the defendant. D. No defense will be successful in this case.

D. No defense will be successful in this case.

A man bought a barbecue grill from a local store. The store only sold barbecue grills and did not manufacture them. One week later, the man noticed that the hose that ran from the propane tank had worn thin and the grill did not heat up as quickly as it should. The man was annoyed at this development, but he had a backyard full of hungry guests and continued to cook. One hour after he first noticed the worn hose, the man began to smell propane. He continued to cook, because he had only one hamburger left on the grill. Just as the man was finishing, the hose burst, and propane began rushing out of the tank. The propane ignited, seriously burning him. Will the man prevail in a strict liability action against the store? A. Yes, because the store employees may have improperly assembled the grill. B. Yes, if the defect in the hose could have been discovered by a reasonable inspection by the store. C. No, because the man was contributorily negligent. D. No, because the man knew that the hose was worn.

D. No, because the man knew that the hose was worn.

The defendant, a blasting company, is preparing to excavate rock by using dynamite and other explosives. Before setting off the explosives, the defendant placed a number of signs around the area warning of the danger of the blasting activities. Despite seeing the signs, the plaintiff entered onto the land near where the explosives had been placed. The plaintiff was seriously injured when the defendant set off the explosives. Will the defendant likely be found liable to the plaintiff for his injuries? A. Yes, because the defendant was engaged in an abnormally dangerous activity. B. Yes, because the plaintiff's conduct may mitigate the defendant's liability, but it will not eliminate it. C. No, because the defendant did not act negligently in conducting the blasting. D. No, because the plaintiff knew of the danger of entering onto the land and voluntarily entered onto the land despite the danger.

D. No, because the plaintiff knew of the danger of entering onto the land and voluntarily entered onto the land despite the danger.

George liked to hit out golf balls at a local driving range. Since he did not own any clubs, he used one of the loaner clubs that the range provided for its customers. One day, George was examining the clubs in the loaner club bag, deciding which drive to use. Sally, who was standing next to George, picked a club out of the bag. Sally was an aspiring golfer and decided to clown around by hitting two golf balls at the same time. Unfortunately, one of the balls ricocheted off of the club at an odd angle and hit George in the mouth, breaking his front teeth. George required over $5,000 in dental work. If George sues the driving range for damages, should he recover? A. Yes, because Sally was an invitee on the premises. B. Yes, because a jury may use the doctrine of res ipsa loquitur to infer a breach of duty on the part of the driving range. C. No, because the George was a licensee on the premises. D. No, because there is no evidence that the driving range failed to exercise reasonable care in keeping the premises safe.

D. No, because there is no evidence that the driving range failed to exercise reasonable care in keeping the premises safe.

Dr Denny, a GP, is sunbathing on Jones Beach when he notices a woman, Pauline, struggling in the water slightly offshore. He jumps into the water and swims out to rescue her. Pauline, however, belongs to a traditionalist religious sect that believes that God determines our destiny in all things, and that it is deeply sinful to interfere with His will. In her view, accepting rescue from an emergency situation like this is deeply sinful and might condemn her to hell. And, as the Book of Mark says, what does it profit a man to gain the whole world if he lose his soul? Accordingly, from the moment when she sees him, Pauline -- who is panicked and has swallowed quite a bit of seawater -- tries to tell him to leave her alone. Dr Denny either does not understand what she is saying or concludes that she is hysterical and doesn't mean it (the testimony is inconsistent). She passes out. He grabs her by the waist and heroically brings her to shore, where he performs mouth to mouth resuscitation and revives her. Once revived, Pauline is furious at having lost her chance of eternal life and somewhat worried that her community will ostracize her (or worse) for having been touched (mouth-to-mouth!) by a strange man. She sues Dr Denny for battery. A. Dr Denny should prevail because, on balance, he did more good than harm to Pauline. B. Dr. Denny should prevail because Pauline's beliefs are unreasonable. C. Pauline should prevail because doctors may not touch a patient without the patient's consent, even if the touching would save the patient's life. D. Pauline should lose because reasonable people would not view touching a drowning stranger to rescue her as "harmful or offensive."

D. Pauline should lose because reasonable people would not view touching a drowning stranger to rescue her as "harmful or offensive."

The Surgeon General determines that weezles and sneezles disease ("W&S") is safe but unpleasant for most children who get it but a small percentage of children will suffer serious complications and could even die. Additionally, W&S is quite dangerous to adults, often causing scarring and sterility and, less often, death. The SG further determines that a vaccination is available which will protect virtually everyone from the disease and its possible complications. People with compromised immune systems (including from cancer treatments) cannot take the vaccination, however. Also, the vaccination itself has complications, some serious, in a very small percentage of cases. The rate of complications from the vaccination is less than 1% of the rate of complications from W&S. Accordingly, the SG requires every schoolchild to receive the vaccination absent good medical cause. Robin, a child, takes the vaccination, over the objections of herself and and her parents, Winnie and Christopher. Unfortunately, Robin suffers serious complications. She sues the Surgeon General for requiring the vaccination. The government has waived any claims of sovereign immunity and has fully indemnified the SG for all lawsuits. Robin maintains, and the SG does not dispute, that if she had not taken the vaccination she would not have suffered the complications - and, since virtually every other child in her vicinity was vaccinated, it is highly unlikely that she would have gotten W&S even without the vaccination. Which of the following is most accurate: A. Robin has a cause of action in battery because the SG intentionally (if indirectly) touched her in way that caused injury and because unconsented medical treatment is always considered offensive or harmful. B. Robin has a cause of action in battery because the vaccination program hurt her to benefit others, and it would be a fundamental violation of her rights to fail to compensate her for what is - in effect - public use of her person. C. Robin has no cause of action in battery because her injury was caused by the existence and danger of W&S, not by any act of the SG. D. Robin has no cause of action in battery because reasonable people would not regard the vaccination as dangerous despite the known possibility of injury.

D. Robin has no cause of action in battery because reasonable people would not regard the vaccination as dangerous despite the known possibility of injury.

Sam was walking down the street, when she saw Paul and Theo in a fight outside a bar. Paul was punching Theo, who was clearly taking it quite badly and looked like he was in danger of serious injury. Sam immediately went to Theo's rescue, clobbering Paul with a stick until he stopped fighting. Paul sues Sam for battery. The evidence shows that Sam had never met either Paul or Theo before and had no idea which of them had started the fight or with what provocation. A. Paul should prevail because Sam hit him in a harmful way, intending to injure him, and caused damage. B. Sam should prevail in a dual intent state provided that she can establish that she intended to stop the fight, not to harm Paul. C. Paul should prevail because Sam is not privileged to defend a stranger. D. Sam should prevail because she responded in a proportional fashion to defend Theo from a battery.

D. Sam should prevail because she responded in a proportional fashion to defend Theo from a battery.

An attorney purchased a brand new car from a dealership. About a month later, the attorney noticed that the manufacturer had just released a new model of the same car. The new model had a much bigger engine and 20% more horsepower. The attorney wanted to upgrade to the new model so he sold the car to his neighbor. The next day, the engine exploded and injured the neighbor and his 4-year-old son. The neighbor filed suit against the attorney. Who should prevail? A. The neighbor, because the attorney is liable for damages stemming from the unreasonably dangerous car that he sold. B. The neighbor, because he can prevail against any of the attorney, the dealership, or the manufacturer. C. The attorney, because he did not cause the defect. D. The attorney, because he was not in the business of selling cars.

D. The attorney, because he was not in the business of selling cars.

A builder specialized in the renovation of old buildings. His most recent project was the conversion of a housing project into luxury condominiums. One of the buildings had been renovated by the city 15 years ago when steel guard rails had been installed on all the windows to prevent children from accidental falls. The builder did most of the exterior painting and brick repair work himself to save money. He greatly improved the appearance of the old buildings by painting the window guards an attractive decorative garden pattern. When renovations were completed, members of the local fire department inspected the premises. The fire inspector was a large, heavy-set man. For public relations and safety purposes, he conducted his inspections in full uniform—big steel-toed black boots, a hard hat, tool belt, keys, and a heavy lined coat. The uniform added 25 pounds to his weight of 225 pounds. The builder showed the inspector around the building. As part of his inspection, the inspector climbed on the window bars to evaluate this means of egress from the apartments on the upper floors in the event of a fire or emergency. When he reached the third floor, the painted window guards gave way, causing the inspector to fall and seriously hurt himself. Examination revealed that underneath the new paint, the window guards was badly corroded and could not support the inspector in his uniform. The inspector sued the builder for his injuries. In a majority jurisdiction, who should prevail? A. The inspector, because the builder received an economic benefit from his inspection of the building. B. The inspector, because the builder had a duty to inspect the window bars. C. The builder, because he did not intentionally conceal the defective bars with paint. D. The builder, because he was not aware of the dangerous condition of the window bars.

D. The builder, because he was not aware of the dangerous condition of the window bars.

A swimmer suffered a sprained ankle, so he went to visit a chiropractor. The chiropractor explained that the fastest way to heal the ankle was to treat it with a new anti-inflammatory medication. The swimmer told the chiropractor that he needed to be ready for an important race, to qualify for the Olympics, in one month. The medication was known to cause dizziness and sometimes nausea. The medicine was considered a supplement and was available at most drug stores. During a course of treatment consisting of two capsules per day for three weeks, the swimmer looked up the medication on the internet. He found an article from a reputable medical journal describing the dangers of using the medication that the chiropractor recommended. In particular, he found that the medicine could cause an inner ear condition that would likely prevent the swimmer from swimming in the future. The swimmer discontinued treatment immediately and was not able to swim in the swim meet because his ankle was still sore. The swimmer went to a physician, who recommended no swimming for six weeks and four months of physical therapy. When the therapy was completed, the swimmer resumed his training for the Olympics. The swimmer brought a suit against the chiropractor. Who should prevail? A. The swimmer, because he would not have consented to the use of the medication if he knew about the risks. B. The swimmer, if a reasonable person would have considered the information about potential side effects important. C. The chiropractor, if she weighed the risks and benefits of using the medication before prescribing it. D. The chiropractor, because the swimmer suffered no harm from the use of the medication.

D. The chiropractor, because the swimmer suffered no harm from the use of the medication.

A man's dream was to open a movie theater that would show nothing but the classics of the 1930s and 1940s - especially musicals. He began looking for an appropriate building, erected during that era, in the heart of downtown. The man soon received a call from a theater owner offering to lease a 150-seat theater downtown to the man. The owner informed the man from the outset that he was willing to lease the building at a discount because some of the overhead lighting was in need of repair. It was not adequately secured; in fact, some had fallen during a recent earthquake, and the owner knew that it would take at least two months to make the necessary repairs. The man signed a 1-month lease, hired a local contractor to repair the lighting, and began an advertising campaign with plans to open for business one month later. Response to the man's concept was overwhelming, with the first ten showings sold out before the opening. The morning of opening day, the contractor informed the man that the lighting would not be completed for at least an additional two days. The man reluctantly proceeded with the opening anyway. The day after the opening, a 5.0 aftershock hit the city. A section of the lighting hanging from the balcony fell, causing head injuries to a customer at one of the showings. If the customer sues the owner, who will prevail? A, The owner, because the man became solely liable for the condition of the premises as of the date he took possession. B. The owner, because he disclosed the condition of the lighting to the man at the time of the lease. C. The customer, because she was an invitee. D. The customer, because the owner knew the man planned to use the building as a theater.

D. The customer, because the owner knew the man planned to use the building as a theater.

A girl and her friend were two 12-year-old classmates. They worked everyday after school on a giant jigsaw puzzle. When the puzzle was finally finished, the girls could not wait to take it to the hobby shop to have it framed. Both wanted to go to the shop to select the frame and the matting. The girls got on a scooter and headed for the hobby shop; the scooter was unstable and wobbled down the block. A woman was driving her new hybrid car toward the girls. She was texting her best friend about how she was saving the planet while driving, and she was not paying attention to the road. The two girls saw her coming and soon realized there was no way to avoid being hit. The friend, overcome with panic, would not respond when the girl urged her to jump off the scooter and into the grass. The girl then pushed her friend off the scooter at the last minute. The driver hit the girl, causing severe head injuries and lacerations. The car did not hit the friend, but she suffered a broken leg and contusions when the girl pushed her off the scooter. The friend's parents, on behalf of their daughter, sue the driver for damages. Who should prevail? A. The parents, unless the driver was driving at the legal speed limit. B. The parents, unless the girl steered the scooter negligently. C. The driver, unless the driver had the last clear chance to avoid the collision. D. The driver, unless the driver was driving negligently when the collision occurred.

D. The driver, unless the driver was driving negligently when the collision occurred.

A college marching band travelled from city to city with the football team. The band travelled on their own bus. The band was known for years for its weekly concerts that were followed by its weekly beer parties on the bus. One of the customs of the band was that all new members had to "run the gauntlet". This was a school tradition, common among all bands on campus. The new members would start at the front of the bus and try to get to the back of the bus for a celebratory beer as their reward. On the way, band members would try to impede new members' progress by hitting the new members with drum sticks. Over the years, no serious injuries were reported. One night, a new member died after being struck in the head by the drum major. As it turns out, the new member had an undiagnosed brain aneurysm. The new member's family sued the drum major. Who should prevail? A. The drum major, because he was acting consistently with a band custom that had gone on for years. B. The drum major, because the new member had an undiagnosed brain aneurysm. C. The family, because the drum major's actions caused the death of the new member. D. The family, because the drum major breached a duty of reasonable care.

D. The family, because the drum major breached a duty of reasonable care.

A teenage girl was getting ready for her spring formal. She had music playing and candles burning while she was putting on her make-up and doing her hair. Earlier that day she had bought some new make-up and a can of hairspray for the big event. The hairspray manufacturer had put far too much alcohol in it, which caused the spray to catch fire too easily. The can plainly read: "DO NOT USE AROUND OPEN FLAME." Nevertheless, the girl proceeded to spray her hair while standing near the candles she had burning. The spray ignited and the girl's long hair went up in flames and her head, shoulders, and face were seriously burned. The girl was rushed to the hospital. Which of the following is correct? A. The girl should not recover, because the can had a warning against spraying near an open flame. B. The girl should not recover, because a reasonable person would not have used the spray while having candles burning. C. The girl should recover, because the warning was inadequate. D. The girl should recover, because the product was defective.

D. The girl should recover, because the product was defective.

A husband and a wife were always misplacing their keys. One day, the husband hung a key rack at eye level just inside the front door of their home. The husband and the wife agreed that each night, when they returned home from work, they would place their keys on the key rack so that each knew at all times where their keys were located. One Sunday afternoon, the husband and the wife decided to walk to the local deli for lunch. Because their 14-year-old daughter was going to be home, they decided they did not need to take their keys, and so they left them hanging on the key rack. Shortly after the husband and the wife left, the daughter decided that she wanted to attempt to drive the family car around the block before her parents returned. Even though she knew she was forbidden from doing so, she removed the keys to the family car off of the key rack, started the family car, and attempted to drive it around the block. However, due to the fact that she was an inexperienced driver, she quickly lost control of the car and crashed into a parked car owned by a neighbor. The car suffered substantial damage. If the neighbor sues the husband and the wife, what will be the most likely result? A. The neighbor will win, because the husband and the wife are liable for the negligence of their daughter. B. The neighbor will win, because a child operating a motor vehicle is an unusually dangerous activity. C. The husband and the wife will win, because a parent is not liable for the tortious acts of a child. D. The husband and the wife will win, unless they did not act reasonably in permitting their daughter access to the car keys.

D. The husband and the wife will win, unless they did not act reasonably in permitting their daughter access to the car keys.

A builder specialized in the renovation of old courthouse buildings. His newest project was the conversion of an 18th century equity court building into a suite of lofts. The building had been updated 25 years ago when a three-story elevator had been installed. The builder was doing much of the repair work himself to save money. He greatly improved the appearance of the old building by painting the exterior an attractive brick red. Rather than spend the money on a licensed elevator repair professional, he personally checked and lubricated the elevator cables to try to eliminate a grinding noise that he noticed when the elevator operated. When renovations were finished, members of the building department were called to inspect the premises and issue a certificate of occupancy. The building inspector was a large, heavy-set man. The builder showed the inspector around the building. As part of his inspection, the inspector took the elevator to the fourth floor. When the elevator reached the second floor, the elevator cable snapped, causing the building inspector to fall and seriously hurt himself. Examination of the elevator cable revealed that because it was improperly lubricated, it slipped off the main gear and could not support the weight of the building inspector. If the inspector sues the builder for his injuries, who will prevail? A. The builder, because he did not intentionally conceal a defect in the elevator cable. B. The builder, because he was not aware of the dangerous condition of the elevator. C. The inspector, because the builder had a duty to inspect the elevator cable. D. The inspector, because the builder was negligent.

D. The inspector, because the builder was negligent.

Dora, 8 years old, recently learned to ride a bike without training wheels. Riding down a suburban sidewalk with her parents trailing her, she saw a rock in front of her. She tried to avoid the rock but instead found herself heading straight towards it. Panicking, she slammed on her brakes, flipped the bicycle and went flying into Petra, who suffered a broken bone. Petra sues for negligence. Plaintiff seeks an instruction that every person owes the duty of acting with the care that a reasonable person would under the circumstances and that a reasonable person is a person of ordinary judgment and knowledge, without regard to the alleged tortfeasor's capacity to control his actions. Which of the following is most correct? A. The judge should give the instruction, which correctly sets out the reasonable person standard for people with diminished capacities such as children. B. The judge should instruct the jury that children are responsible for the consequences of their actions when they participate in adult activities. C. The judge should instruct the jury that when a child engages in an activity that is inherently dangerous, the child should be held to an adult standard of care, which means acting with the care that a reasonable adult would under the circumstances. D. The judge should instruct the jury that it is the duty of a child to act with the same care that a reasonably careful child of the same age, maturity, training and experience, would act under similar circumstances.

D. The judge should instruct the jury that it is the duty of a child to act with the same care that a reasonably careful child of the same age, maturity, training and experience, would act under similar circumstances.

The plaintiff owned four motorcycles and worked on them every weekend. Needing brakes for one of his motorcycles, the plaintiff took his bike to a mechanic that his buddy had previously recommended. The plaintiff watched very carefully while the mechanic installed the new brakes, and he felt that the mechanic did a great job. However, the plaintiff's brakes failed on the way home from the garage and he crashed into a tree, breaking his leg. An analysis later discovered that the brake parts the mechanic used were a unique style that required higher positioning than normal. The mechanic had undergone the manufacturer's training about installing the unique-style brakes, but when he installed the parts on the plaintiff's bike, he simply positioned the brakes in the usual place well known to all good mechanics, including the plaintiff. Which of the following is correct? A. The mechanic should not be held liable for negligence, because the plaintiff watched the mechanic do the work. B. The mechanic should not be held liable for negligence, because he did what most good mechanics would have viewed as an excellent brake job. C. The mechanic should be held liable for negligence, because he used the unique-style brake parts. D. The mechanic should be held liable for negligence, because he had undergone training about installing the unique-style brake parts.

D. The mechanic should be held liable for negligence, because he had undergone training about installing the unique-style brake parts.

In Snyder v. Turk, Dr. Turk moved for a directed verdict -- that is, that the court determine that no reasonable jury could hold for Ms. Snyder -- on the ground that there was no evidence he "intended to inflict personal injury". The Trial court accepted this argument. The Court of Appeals disagreed. Which is the best summary of the disagreement between the two courts? A. The Trial court believed that no reasonable jury could find that Ms. Snyder was harmed. The Appeals court points out that "offense" is sufficient, without physical harm. B. The Trial court believed that no evidence supported the claim that Dr. Turk "intended" to offend. The Appellate Court points out that a jury could infer such intent from his conduct. C. The Trial Court believed that Dr. Turk's actions were not offensive. The Appellate court points out that a jury could reasonably conclude that a reasonable person would be offended by them. D. The opinion as we have it does not distinguish between the above explanations.

D. The opinion as we have it does not distinguish between the above explanations.

A security guard at a small clothing store told a man and a woman to leave the store as it was closing time. Instead of leaving, the man and woman went to the back of the store to do some last minute shopping. The security guard did not realize the man and woman were still in the store and he locked the doors to the store and left for the night. The woman took her selections to the dressing room to try them on, while the man said on a bench in the waiting area and dozed off. Shortly thereafter, the woman realized they were locked in the store and called the police to come and open the doors for them to leave. By the time the man woke up, the police had already come and unlocked the doors. Which of the following statements is true? A. The security guard will be liable for false imprisonment as to the woman, because she knew she was confined to the store, but not as to the man, because he was unaware that he was locked in the store. B. The security guard will be liable for false imprisonment as to both the man and the woman, because both were confined in the store against their will. C. The security guard will be liable for false imprisonment as to both the man and the woman, because he was negligent in checking the store for customers before locking the doors. D. The security guard will not be liable for false imprisonment as to the man and the woman, because he did not have the requisite intent for false imprisonment.

D. The security guard will not be liable for false imprisonment as to the man and the woman, because he did not have the requisite intent for false imprisonment.

A caterer negligently used spoiled ingredients to prepare food for a wedding, and several guests who consumed the food fell violently ill with food poisoning. The guests called an ambulance to transport them to a hospital. On the way to the wedding to pick up the guests, the driver of the ambulance negligently ran a red light. The ambulance driver caused an accident with a motorist, injuring himself, another EMT worker who was riding in the ambulance, and the motorist. The caterer will be liable for the injuries to whom? A. The wedding guests only. B. The wedding guests and the ambulance driver. C. The wedding guests and the EMT worker. D. The wedding guests, the ambulance driver, the EMT worker, and the motorist.

D. The wedding guests, the ambulance driver, the EMT worker, and the motorist.

A retiree loved his rose garden. He spent many hours tending to his roses and rejoiced when they were in bloom. One afternoon while the roses were blooming, a pedestrian was walking along the sidewalk next to the retiree's property when a car came speeding down the street. The driver of the car momentarily lost control of his vehicle and headed right for the pedestrian. To avoid being hit by the car, the pedestrian jumped onto the retiree's land, knocking several blooms off of one of the retiree's rose bushes. The retiree happened to be leaving his front door at that moment. He did not see the speeding car, but he did see the pedestrian jump onto his land and damage his rose bush. Incensed, the retiree picked up a rock and threw it at the pedestrian. The rock sailed past the pedestrian less than an inch from his face. Can the pedestrian recover from the retiree for assault? A. No, because the pedestrian was not touched by the rock. B. No, because the pedestrian was a trespasser. C. Yes, because the retiree used excessive force. D. Yes, because the pedestrian must have seen the rock pass in front of him.

D. Yes, because the pedestrian must have seen the rock pass in front of him.

Dorrie, an undergraduate, decides it would be very funny to secretly photograph his girlfriend, Petunia, while they are having sex and then share the pictures with his friends. He does so and they, in turn, distribute them all over campus. Petunia learns of what he has done when she hears people calling her the "Porn Queen." She is deeply hurt, so much so that she gets physically ill, is unable to leave her room for a week, and has to leave school. Petunia would like to sue Dorrie for intentionally violating her dignity, privacy and self respect. Assuming the facts above are undisputed and that no other facts are in evidence, In which of the following tort claims is she most likely to prevail? A. Battery B. Assault C. False Imprisonment D. Intentional Infliction of Emotional Distress E. None of the Above

E. None of the Above


Related study sets

Property/Casualty CH.6 EXAM QUESTIONS

View Set

Africa's Climate & Vegetation Reading Notes (Ch. 18.2)

View Set

Starting Out with Python, 2e Chapter 13 (Ch.12)

View Set